Location via proxy:   [ UP ]  
[Report a bug]   [Manage cookies]                
Download as pdf or txt
Download as pdf or txt
You are on page 1of 95

All rights reserved.

No part of Shoulder and Elbow Self-Assessment Examination may be reproduced,


stored in a retrieval system, or transmitted in any form or by any means (electronic, mechanical,
photocopying, recording, or otherwise) without the prior written permission of the publisher.

Published January 2014


American Academy of Orthopaedic Surgeons
6300 North River Road
Rosemont, IL 60018

Copyright© 2014 by the American Academy of Orthopaedic Surgeons

Requests for permission to reproduce any part of the work should be mailed to:
Attention: Examinations Department
American Academy of Orthopaedic Surgeons
6300 North River Road
Rosemont, IL 60018

Printed in the USA

© 2014 American Academy of Orthopaedic Surgeons 2014 Shoulder and Elbow Self-Assessment Examination
2014 Shoulder and Elbow Self-Assessment Examination Answer Book• 11

Question 1
A 45-year-old man who had gout in his foot 2 years ago has a 3-day history of elbow pain without an
injury. The pain is diffuse, constant, and worse with any movement. Examination shows motion from 20
degrees to 90 degrees. There is no erythema around his elbow, he has no fever, and a sensory and motor
examination of his arm is unremarkable. Radiographs only show an effusion. The patient's uric acid level
is within defined limits. What is the next diagnostic step?

1. Elbow joint aspiration


2. MRI scan
3. Splint for 2 weeks and repeat examination
4. Sedimentation rate and C-reactive protein level

PREFERRED RESPONSE: 1

DISCUSSION
The best way to make the diagnosis is to aspirate the fluid from the joint and send it to the laboratory
for a cell count and crystal search. This will allow for the diagnosis of an infection, gout, or pseudogout.
An MRI scan will confirm the examination finding of an effusion, but it will not reveal the cause of an
inflammatory effusion. If the patient has chronic gout, an MRI scan or ultrasound can aid in diagnosis if
gout tophi are seen. A splint will help relieve the pain from the effusion and the effusion may resolve on its
own, but it will not contribute to a diagnosis. Sedimentation rate and C-reactive protein level will help to
rule out an infection, but they are not as diagnostic as an aspiration.

RECOMMENDED READINGS
Orzechowski NM, Mason TG. Seronegative inflammatory arthritis. In: Morrey BF, Sanchez-Sotelo J, eds.
The Elbow and Its Disorders. 4th ed. Philadelphia, PA: Saunders-Elsevier; 2009:1039-1041.

de Avila Fernandes E, Kubota ES, Sandim GB, Mitraud SA, Ferrari AJ, Fernandes AR. Ultrasound
features oftophi in chronic tophaceous gout. Skeletal Radiol. 2011 Mar;40(3):309-15. Epub 2010 Jul 31.
PubMed PMID: 20676636.

© 2014 American Academy of Orthopaedic Surgeons 2014 Shoulder and Elbow Self-Assessment Examination
12 • American Academy of Orthopaedic Surgeons

Figure 2

Question 2
A 65-year-old man who underwent an uncomplicated reverse total shoulder arthroplasty (rTSA) to treat
rotator cuff arthropathy 2 years ago has a routine follow-up visit in your clinic. A radiograph at 2-year
followup is shown in Figure 2. He denies shoulder pain and dysfunction and constitutional symptoms,
and his clinical examination findings are benign. Based upon the present radiologic evaluation, what is the
next most appropriate step?

1. Revision rTSA
2. Conversion to hemiarthroplasty
3. Continued observation
4. Infection work-up with screening labs and joint aspiration

PREFERRED RESPONSE: 3

DISCUSSION
Based upon the patient's clinical examination and symptoms, continued observation is appropriate.
The remaining options are not indicated. The radiograph reveals scapular notching, one of the more
common complications specific to rTSA. Notching is caused by repeated contact between the humeral
component and/or humerus and the inferior pillar of the scapular neck. Generation of particulate debris
from this interaction can result in osteolysis with the potential for screw and base plate failure. The overall
incidence of notching has been reported to be between 51% and 96%. This nearly ubiquitous finding has
been attributed to implant positioning, altered glenoid and humeral anatomy, and duration of implantation.
Recent studies that indicate increased lateral offset, increased glenosphere size, and inferior positioning of
the base plate may reduce the incidence of scapular notching.

© 2014 American Academy of Orthopaedic Surgeons 2014 Shoulder and Elbow Self-Assessment Examination
2014 Shoulder and Elbow Self-Assessment Examination Answer Book• 13

RECOMMENDED READINGS
Cheung E, Willis M, Walker M, Clark R, Frankle MA. Complications in reverse total shoulder
arthroplasty. J Am Acad Orthop Surg. 2011 Jul;19(7):439-49. Review. PubMed PMID: 21724923.

Levigne C, Boileau P, Favard L, Garaud P, Mole D, Sirveaux F, Walch G. Scapular notching in reverse
shoulder arthroplasty. J Shoulder Elbow Surg. 2008 Nov-Dec;l 7(6):925-35. Epub 2008 Jun 16. PubMed
PMID: 18558499.

Gutierrez S, Levy JC, Frankle MA, Cuff D, Keller TS, Pupello DR, Lee WE 3rd. Evaluation of abduction
range of motion and avoidance of inferior scapular impingement in a reverse shoulder model. J Shoulder
Elbow Surg. 2008 Jul-Aug;l 7(4):608-15. Epub 2008 Mar 6. PubMed PMID: 18325795.

Question 3
A 61-year-old right-hand-dominant woman sustains a fall down 3 stairs, resulting in a left anteroinferior
dislocation and noncomminuted greater tuberosity fracture. A closed glenohumeral reduction with
intravenous sedation is performed in the emergency department. After reduction, the greater tuberosity
fragment remains displaced by 2 mm. What is the most appropriate treatment?

1. Open reduction internal fixation with transosseous sutures


2. Arthroscopic fixation using a suture bridge technique
3. Nonsurgical treatment with early passive range of motion
4. Nonsurgical treatment with sling immobilization for 4 weeks

PREFERRED RESPONSE: 3

DISCUSSION
Greater tuberosity fractures and rotator cuff tears associated with a traumatic dislocation are more
commonly seen in women older than age 60. Greater tuberosity fractures that are displaced less than 5
mm in the general population and less than 3 mm in laborers and professional athletes can be treated
successfully without surgery. Early passive range of motion is important to avoid the complication of
stiffness.

RECOMMENDED READINGS
George MS. Fractures of the greater tuberosity of the humerus. J Am Acad Orthop Surg. 2007
Oct;l5(10):607-13. Review. PubMed PMID: 17916784.

Platzer P, Kutscha-Lissberg F, Lehr S, Vecsei V, Gaebler C. The influence of displacement on


shoulder function in patients with minimally displaced fractures of the greater tuberosity. Injury. 2005
Oct;36(10):1185-9. Epub 2005 Jun 16. PubMed PMID: 15963996.

Mattyasovszky SG, Burkhart KJ, Ahlers C, Proschek D, Dietz SO, Becker I, Muller-Haberstock
S, Muller LP, Rommens PM. Isolated fractures of the greater tuberosity of the proximal humerus:
a long-term retrospective study of 30 patients. Acta Orthop. 2011 Dec;82(6):714-20. doi:
10.3109/17453674.2011.618912. Epub 2011 Sep 6. PubMedPMID: 21895502.

© 2014 American Academy of Orthopaedic Surgeons 2014 Shoulder and Elbow Self-Assessment Examination
14 • American Academy of Orthopaedic Surgeons

Figure 4a Figure 4a

Question 4
A 30-year-old man with diabetes sustained an acute posterior dislocation of his right shoulder after a
seizure event that required emergency department reduction. You initially treat him with a sling for 4
weeks and then refer him for outpatient therapy. During his therapy sessions, the patient admits to pain
and instability symptoms during range of motion exercises. Repeat examination indicates a positive
posterior load-shift test and apprehension with adduction and internal rotation of the shoulder. His CT and
MRI scans are shown in Figures 4a and 4b. What is the most appropriate next step in treating his injury?

l. Brief period of immobilization and reinitiation of therapy


2. Open posterior capsular shift with labral repair
3. Arthroscopic capsulolabral repair with subscapularis and lesser tuberosity transfer
4. Resurfacing arthroplasty with labral repair

PREFERRED RESPONSE: 3

DISCUSSION
Posterior glenohumeral dislocations are much less common than anterior glenohumeral dislocations,
with a prevalence of 1.1 per 100,000 per year. There is a bimodal distribution with a peak in young men
(2.4 men to 1 woman) and a second peak in elderly people with a more equivalent gender ratio. Posterior
dislocations most commonly result from trauma, with the remainder of events secondary to seizure
activity. According to Robinson and associates, age younger than 40 years, dislocation during a seizure,
and a large Hill-Sachs lesion were all predictive of recurrent instability. Concomitant injuries associated
with posterior shoulder dislocations include capsulolabral tears, fractures, and rotator cuff tears. Imaging
studies in this patient indicate a reverse Hill-Sachs lesion with a corresponding posterior labral tear.
Because of his persistent mechanical symptoms, continued immobilization and therapy is not appropriate.
An open capsular shift with labral repair alone would not address symptoms related to an engaging Hill-
Sachs lesion. Based upon the patient's age, activity level, and percentage of humeral head involvement,
a resurfacing arthroplasty is not recommended. Historically, the Zuckerman procedure, lesser tuberosity
transfer, has been used to address symptomatic reverse Hill-Sachs lesions (20% to 40% humeral head
involvement) associated with posterior shoulder dislocations. Modifications of this technique such as
arthroscopic transfer of the subscapularis tendon without posterior capsulorrhaphy have proven beneficial.

© 2014 American Academy of Orthopaedic Surgeons 2014 Shoulder and Elbow Self-Assessment Examination
2014 Shoulder and Elbow Self-Assessment Examination Answer Book• 15

RECOMMENDED READINGS
Rouleau DM, Hebert-Davies J. Incidence of associated injury in posterior shoulder dislocation: systematic
review of the literature. J Orthop Trauma. 2012Apr;26(4):246-51. Review. PubMed PMID: 22183196.

Robinson CM, Seah M, Akhtar MA. The epidemiology, risk of recurrence, and functional outcome after
an acute traumatic posterior dislocation of the shoulder. J Bone Joint SurgAm. 2011 Sep 7;93(17):1605-
13. PubMed PMID: 21915575.

Krackhardt T, Schewe B, Albrecht D, Weise K. Arthroscopic fixation of the subscapularis tendon in the
reverse Hill-Sachs lesion for traumatic unidirectional posterior dislocation of the shoulder. Arthroscopy.
2006 Feb;22(2):227.el-227.e6. PubMed PMID: 16458812.

CLINICAL SITUATION FOR QUESTIONS 5 AND 6


A 40-year-old riveter who works in a manufacturing plant experiences gradual onset of anterior shoulder
pain that started 4 months ago, and he now has a constant ache in his shoulder. His pain wakes him at
night and is worsened by lifting anything at shoulder height. He does not have any radiation of pain, and
neck motion does not aggravate his pain. He stopped doing riveting work 1 month ago, but the pain did
not improve. Examination shows normal motion and strength, a positive impingement sign, and tenderness
over the anterior greater tuberosity. His sulcus sign is negative, and his Yergason and speed test :findings
are negative. He has normal scapular mechanics.

Question 5
What is the next most appropriate step to confirm the diagnosis?

1. Radiograph
2. MRI scan
3. Subacromial injection with lidocaine
4. Ultrasound

PREFERRED RESPONSE: 3

© 2014 American Academy of Orthopaedic Surgeons 2014 Shoulder and Elbow Self-Assessment Examination
16 • American Academy of Orthopaedic Surgeons

Figure 5

Question 6
He started physical therapy while continuing light duty at work. Eight weeks later, his pain remained
unchanged. An MRI scan is shown in Figure 5. What histologic changes are likely to be found in the
supraspinatustendon?

1. Disorganized collagen fibers with mucoid degeneration


2. Disorganized collagen fibers and acute inflammatory cells
3. Normal tendon fibers infiltrated with capillary proliferation
4. Normal tendon fibers infiltrated with acute inflammatory cells

PREFERRED RESPONSE: 1

DISCUSSION FOR QUESTIONS 5 AND 6


This patient has impingement syndrome based on the history and examination. The best way to confirm
the diagnosis is by performing a subacromial injection with lidocaine, which is also called a Neer
impingement test. If the pain is relieved, the patient's pain is coming from the subacromial space. An
MRI scan would not confirm the diagnosis of impingement, although it can aid in diagnosis of other
causes of anterior shoulder pain such as a rotator cuff tear. This patient has normal rotator cuff strength,
so that diagnosis is less likely. A radiograph can show acromial morphology, which would support the
diagnosis of impingement, but it does not rule out impingement if the radiograph findings are normal.
Ultrasound would not support the diagnosis of impingement, but, like an MRI scan, it can reveal
pathologies other than impingement. The MRI scan shows a supraspinatus tendon with changes consistent
with tendinopathy, which is defined by disorganized collagen fibers with mucoid degeneration on the
microscopic level. Although there are always exceptions, most tendinopathy occurs without inflammatory
cells or capillary proliferation.

© 2014 American Academy of Orthopaedic Surgeons 2014 Shoulder and Elbow Self-Assessment Examination
2014 Shoulder and Elbow Self-Assessment Examination Answer Book • 17

RECOMMENDED READINGS FOR QUESTIONS 5 AND 6


Park HB, Yokota A, Gill HS, El Rassi G, McFarland EG. Diagnostic accuracy of clinical tests for the
different degrees of subacromial impingement syndrome. J Bone Joint SurgAm. 2005 Jul;87(7):1446-55.
PubMed PMID: 15995110.

Khan KM, Cook JL, Bonar F, Harcourt P, Astrom M. Histopathology of common tendinopathies. Update
and implications for clinical management. Sports Med. 1999 Jun;27(6):393-408. Review. PubMed PMID:
10418074.

Lauder TD. Musculoskeletal disorders that frequently mimic radiculopathy. Phys Med Rehabil Clin N Am.
2002 Aug;13(3):469-85. Review. PubMed PMID: 12380546.

Cannon DE, Dillingham TR, Miao H, Andary MT, Pezzin LE. Musculoskeletal disorders in referrals for
suspected cervical radiculopathy.Arch Phys Med Rehabil. 2007 Oct;88(10):1256-9. PubMed PMID:
17908566.

END OF SERIES

CLINICAL SITUATION FOR QUESTIONS 7 THROUGH 9


A 26-year-old man has the chief complaint of right shoulder instability. He underwent an uncomplicated
arthroscopic Bankart repair following an injury sustained while playing high school football. His condition
was stable for 7 years, but he redislocated his shoulder in a fall 6 months ago. He describes weekly
anterior instability events that he can reduce on his own. Radiographs reveal a located glenohumeral joint,
but a Hill-Sachs lesion is noted. A CT scan reveals a 20% anteroinferior glenoid deficiency and a Hill-
Sachs lesion involving 20% of the humeral head.

Question 7
What is the best surgical option for this patient?

1. Coracoid transfer
2. Open Bankart repair
3. Revision arthroscopic Bankart repair
4. Arthroscopic remplissage procedure

PREFERRED RESPONSE: 1

(t:) 2014 American Academy of Orthopaedic Surgeons 2014 Shoulder and Elbow Self-Assessment Examination
18 • American Academy of Orthopaedic Surgeons

Question 8
What is the best indication to treat a Hill-Sachs lesion?

1. A lesion involving 20% of the humeral head that does not engage on examination
2. A lesion involving 25% of the humeral head that remains located following instability repair
3. A lesion involving 30% of the humeral head that engages on examination
4. A lesion involving 40% of the humeral head with recurrent glenohumeral instability

PREFERRED RESPONSE: 4

Question 9
What is the most likely predictor of postsurgical pain following a coracoid transfer procedure for recurrent
shoulder instability?

1. Suboptimal graft placement


2. Pain before surgery
3. Progression of osteoarthritis
4. Previous surgical treatment

PREFERRED RESPONSE: 2

DISCUSSION FOR QUESTIONS 7 THROUGH 9


Recurrent glenohumeral instability with anteroinferior glenoid bone loss is best treated with a coracoid
transfer. Schmid and associates reported on 49 patients with these lesions and recurrent instability who
underwent coracoid transfer procedures. Good and excellent outcomes were reported in 88% of patients,
and there were no instances of recurrent instability. With anteroinferior glenoid bone loss, capsular
procedures-whether open or arthroscopic-are unlikely to provide stability. Remplissage is a procedure
designed to address Hill-Sachs lesions rather than glenoid defects. The absolute indications to treat Hill-
Sachs lesions surgically include a lesion associated with a humeral head fracture, a lesion involving more
than 30% of the humeral head with chronic instability, and reverse Hill-Sachs lesions involving more than
20% of the articular surface with symptoms of posterior instability. Lesions involving 20% to 35% with
or without engagement on examination are relative indications, as are lesions exceeding 10% that do not
remain centered in the glenoid following arthroscopic stabilization.

In Schmid and associates' series of coracoid transfers for recurrent instability and anterior glenoid
deficiency, patients who reported pain before surgery were 20 times more likely to have pain after surgery
that compromised the functional outcome. Optimal graft placement correlated with better functional
outcomes and less progression of arthrosis, but not with pain. Consequently, poor graft position, arthritis
progression, and prior surgical treatment are not as consistently predictive of pain after surgery.

© 2014 American Academy of Orthopaedic Surgeons 2014 Shoulder and Elbow Self-Assessment Examination
2014 Shoulder and Elbow Self-Assessment Examination Answer Book• 19

RECOMMENDED READINGS FOR QUESTIONS 7 THROUGH 9


Schmid SL, Farshad M, Catanzaro S, Gerber C. The Latarjet procedure for the treatment ofrecurrence
of anterior instability of the shoulder after operative repair: a retrospective case series of forty-nine
consecutive patients. J Bone Joint SurgAm. 2012 Jun 6;94(11):e75. doi: 10.2106/JBJS.K.00380. PubMed
PMID: 22637215.

Provencher MT, Frank RM, Leclere LE, Metzger PD, Ryu JJ, Bernhardson A, Romeo AA. The Hill-Sachs
lesion: diagnosis, classification, and management. J AmAcad Orthop Surg. 2012 Apr;20(4):242-52. doi:
10.5435/JAAOS-20-04-242. Review. PubMed PMID: 22474094.

Boileau P, O'Shea K, Vargas P, Pinedo M, Old J, Zumstein M. Anatomical and functional results after
arthroscopic Hill-Sachs remplissage. J Bone Joint Surg Am. 2012 Apr 4;94(7):618-26. doi: 10.2106/
JBJS.K.00101. PubMed PMID: 22488618.

END OF SERIES

Question 10
A 45-year-old woman with diabetes has a 3-month history of left shoulder pain and motion loss unrelated
to trauma. She previously underwent treatment with nonsteroidal anti-inflammatory medication and a
home stretching program, experiencing minimal relief of her symptoms. Examination reveals loss of
passive external rotation, abduction, and forward elevation without reduction in strength. Radiograph
findings are normal. What is the most appropriate next step?

1. MRI scan
2. Cortisone injection therapy with continued physical therapy (PT)
3. Closed manipulation under anesthesia
4. Arthroscopic release with manipulation under anesthesia

PREFERRED RESPONSE: 2

© 2014 American Academy of Orthopaedic Surgeons 20 I4 Shoulder and Elbow Self-Assessment Examination
20 • American Academy of Orthopaedic Surgeons

DISCUSSION
Based upon the duration of symptoms and clinical presentation, this patient would benefit from cortisone
injection therapy and continued PT. Adhesive capsulitis is most commonly an idiopathic process that
results in joint pain and loss of motion from capsular contracture. It affects approximately 2% to 5% of
the general population. The process typically affects middle-age women. There are secondary causes
such as previous trauma and fractures as well as associated medical conditions such as diabetes, stroke,
and cardiac and thyroid disease. Debate remains as to whether there is a genetic predisposition for
the development of adhesive capsulitis despite increased frequency noted in twin studies. Although
the underlying etiology and pathophysiology are not well understood, the consensus is that synovial
inflammation and capsular fibrosis result in pain and joint volume loss. It is hypothesized that in patients
with diabetes, an increased rate of glycosylation and cross-linking of the shoulder capsule raises the
incidence of frozen shoulder. For this patient, history reveals a short course of symptoms that did not
improve with nonsurgical modalities. Clinically, the patient has reduced passive range of motion,
particularly with external and internal rotation and forward elevation. Radiographs are usually obtained
to exclude other causes of shoulder pain such as glenohumeral arthrosis, malignancy, calcific tendonitis,
impingement, and acromioclavicular degeneration. If pain and stiffness persist beyond 6 months, closed
manipulation may be an option. Complications associated with this modality may include humerus
fracture, dislocation, hematoma, rotator cuff and labral tears, and brachia! plexus injury. Some surgeons
advocate arthroscopic capsular release to allow for examination of concomitant pathology and controlled
release of capsular tissue, with the potential for reduced required force when performing the manipulation
portion of the procedure. This modality may be appropriate after an initial treatment with PT. Controversy
remains as to whether posterior capsular release should be performed routinely because studies have
shown outcomes to be similar with anterior and combined approaches. Therapy should be initiated early
after intervention, with some surgeons advocating admission to the hospital with inpatient therapy for pain
management and compliance.

RECOMMENDED READINGS
Neviaser AS, Neviaser RJ. Adhesive capsulitis of the shoulder. J Am Acad Orthop Surg. 2011
Sep;l9(9):536-42. Review. PubMed PMID: 21885699.

Hsu JE, Anakwenze OA, Warrender WJ, Abboud JA. Current review of adhesive capsulitis. J Shoulder
Elbow Surg. 2011 Apr;20(3):502-14. Epub 2010 Dec 16. Review. PubMed PMID: 21167743.

Le Lievre HM, Murrell GA. Long-term outcomes after arthroscopic capsular release for idiopathic
adhesive capsulitis. J Bone Joint SurgAm. 2012 Jul 3;94(13):1208-16. PubMed PMID: 22760389.

Yian EH, Contreras R, Sodl JF. Effects of glycemic control on prevalence of diabetic frozen shoulder. J
Bone Joint SurgAm. 2012 May 16;94(10):919-23. PubMed PMID: 22617920.

© 2014 American Academy of Orthopaedic Surgeons 2014 Shoulder and Elbow Self-Assessment Examination
2014 Shoulder and Elbow Self-Assessment Examination Answer Book• 21

Question 11
A 42-year-old woman has a posterior elbow dislocation. Closed reduction is performed, and the elbow
appears stable under fluoroscopic examination. Further treatment should consist of

1. early mobilization only.


2. surgical repair or reconstruction of the lateral collateral ligament (LCL) and the medial
collateral ligament (MCL).
3. active range of motion in a hinged brace with a range of 30 degrees to 120 degrees.
4. application of a hinged external fixator with early mobilization.

PREFERRED RESPONSE: 1

DISCUSSION
This is a simple (no associated fracture) elbow dislocation. Such dislocations can be treated with closed
reduction followed by mobilization after 5 to 7 days to avoid stiffness, provided the elbow is stable
through a full arc of motion at the time of reduction. If the elbow is unstable but has a short arc of stability,
then using a hinged brace in the stable arc may be considered. (Note: It may be necessary to splint the
elbow in pronation if the MCL is intact and the LCL is disrupted or in supination if the LCL is intact but
the MCL disrupted.) Surgical reconstruction of the LCL and MCL may be required only if the elbow does
not have a stable arc at the time of reduction. If unstable after reconstruction, application of a hinged
external :fixator may be considered.

RECOMMENDED READINGS
Glaser D, Armstrong A. Elbow and forearm trauma. In: Flynn Jed. Orthopaedic Knowledge Update 10.
Rosemont, IL: American Academy of Orthopaedic Surgeons; 2011:325-341.

Anakwe RE, Middleton SD, Jenkins PJ, McQueen MM, Court-Brown CM. Patient-reported outcomes
after simple dislocation of the elbow. J Bone Joint Surg Am. 2011 Jul 6;93(13):1220-6. doi: 10.2106/
JBJS.J.00860. PubMed PMID: 21776575.

MehlhoffTL, Noble PC, Bennett JB, Tullos HS. Simple dislocation of the elbow in the adult. Results after
closed treatment. J Bone Joint SurgAm. 1988 Feb;70(2):244-9. PubMed PMID: 3343270.

RESPONSES FOR QUESTIONS 12 THROUGH 16


1. Physical therapy and activity modification
2. Corticosteroid injection
3. Arthroscopic glenohumeral capsular release
4. Arthroscopic superior labrum anterior to posterior (SLAP) repair
5. Arthroscopic subacromial decompression and rotator cuff debridement
6. Arthroscopic subacromial decompression and rotator cuff repair

Match the treatment above with the clinical scenario below

© 2014 American Academy of Orthopaedic Surgeons 2014 Shoulder and Elbow Self-Assessment Examination
22 • American Academy of Orthopaedic Surgeons

Question 12
A 65-year-old woman has 4 months of atraumatic shoulder pain persisting despite physical therapy and
activity modification. She has normal range of motion, and an MRI scan reveals a 10% thickness partial
articular supraspinatus tear.

PREFERRED RESPONSE: 2

Question 13
A 35-year-old mechanic has 6 months of shoulder pain following an axial traction work-related injury. His
pain has persisted despite extensive physical therapy and work restrictions. A noncontrast MRI scan shows
a 90% partial bursal-sided supraspinatus tear.

PREFERRED RESPONSE: 6

Question 14
A 25-year-old tennis player has a type II SLAP lesion, with 4 weeks of new-onset atraumatic shoulder
pain.

PREFERRED RESPONSE: 1

Question 15
A 49-year-old woman has 12 months of shoulder pain, global glenohumeral motion loss, and is
nonresponsive to a home stretching program and an intra-articular glenohumeral corticosteroid injection.
MRI scans reveal no full-thickness rotator cuff tears.

PREFERRED RESPONSE: 3

© 2014 American Academy of Orthopaedic Surgeons 2014 Shoulder and Elbow Self-AssessmentExamination
2014 Shoulder and Elbow Self-Assessment Examination Answer Book• 23

Question 16
A 75-year-old man has had 8 months of persistent, atraumatic shoulder pain. He had transient
improvement with physical therapy and a subacromial corticosteroid injection. MRI scan shows a 25%
partial articular supraspinatus/subscapularis tear and significant subacromial bursa! inflammation.

PREFERRED RESPONSE: 5

DISCUSSION FOR QUESTIONS 12 THROUGH 16


The management of partial rotator cuff tears depends upon many factors, including patient age, symptom
onset and severity, prior treatment, physical limitation(s) based on history and examination, and the extent
of structural involvement based upon detailed imaging (typically MRI or ultrasound). The incidence of
a partial rotator cuff tear, based on imaging with MRI or ultrasound, is high in patients ages 60 years
and older. Patients beyond age 60 with either mild or new-onset symptoms with preserved active and
passive range of motion are excellent candidates for physical therapeutic intervention and avoidance
of exacerbating activities, particularly when MRI scan or ultrasound reveal less than 50% tendon
involvement. Partial rotator cuff tears are also common in the dominant arm of overhead athletes, and
frequently respond to nonsurgical treatment, as well. These types of partial rotator cuff tears often are
seen in combination with superior labral pathology. Rotator cuff repair usually is recommended for
patients with tears that involve more than 50% of tendon thickness who have failed a reasonable attempt
at nonsurgical management, particularly patients who are young and have high activity demands. Partial-
sided bursa! tears may be more symptomatic and respond well to surgical repair, but patients may not
do as well with subacromial decompression alone. Global loss of glenohumeral motions is consistent
with adhesive capsulitis. Such patients are initially treated with therapy that emphasizes range of motion,
usually incorporating a home exercise program. Finally, subacromial decompression may be considered
for patients with low-grade partial articular rotator cuff tears that have failed nonsurgical management and
substantially interfere with daily and/or recreational activities.

© 2014 American Academy of Orthopaedic Surgeons 2014 Shoulder and Elbow Self-Assessment Examination
24 • American Academy of Orthopaedic Surgeons

RECOMMENDED READINGS FOR QUESTIONS 12 THROUGH 16


Sher JS, Uribe JW, Posada A, Murphy BJ, Zlatkin MB. Abnormal :findings on magnetic resonance images
of asymptomatic shoulders. J Bone Joint SurgAm. 1995 Jan;77(1):10-5. PubMed PMID: 7822341.

Weber SC. Arthroscopic debridement and acromioplasty versus mini-open repair in the treatment of
significant partial-thickness rotator cuff tears. Arthroscopy. 1999 Mar;15(2):126-31. PubMed PMID:
10210067.

Cordasco FA, Backer M, Craig EV, Klein D, Warren RF. The partial-thickness rotator cuff tear: is
acromioplasty without repair sufficient? Am J Sports Med. 2002 Mar-Apr;30(2):257-60. PubMed PMID:
11912097.

Connor PM, Banks DM, Tyson AB, Coumas JS, D'Alessandro DF. Magnetic resonance imaging of the
asymptomatic shoulder of overhead athletes: a 5-year follow-up study. Am J Sports Med. 2003 Sep-
Oct;31(5):724-7. PubMed PMID: 12975193.

Yamaguchi K, Ditsios K, Middleton WD, Hildebolt CF, Galatz LM, Teefey SA. The demographic and
morphological features of rotator cuff disease. A comparison of asymptomatic and symptomatic shoulders.
J Bone Joint Surg Am. 2006 Aug;88(8):1699-704. PubMed PMID: 16882890.

Loeffler BJ, Brown SL, D'Alessandro DF, Fleischli JE, Connor PM. Incidence of False Positive Rotator
Cuff Pathology in MRis of Patients with Adhesive Capsulitis. Orthopedics. 2011 May 18;34(5):362. doi:
10.3928/01477447-20110317-14. PubMed PMID: 21598899.

Pedowitz RA, Yamaguchi K, Ahmad CS, Burks RT, Flatow EL, Green A, Iannotti JP, Miller BS, Tashjian
RZ, Watters WC 3rd, Weber K, Turkelson CM, Wies JL, Anderson S, St Andre J, Boyer K, Raymond
L, Sluka P, McGowan R; American Academy of Orthopaedic Surgeons. Optimizing the management of
rotator cuff problems. J Am Acad Orthop Surg.2011 Jun;19(6):368-79. PubMed PMID: 21628648.

END OF SERIES

© 2014 American Academy of Orthopaedic Surgeons 2014 Shoulder and Elbow Self-AssessmentExamination
2014 Shoulder and Elbow Self-Assessment Examination Answer Book• 25

Figure 17

Question 17
The fracture seen in Figure 17 is most likely associated with injury to what ligamentous structure?

1. Inferior glenohumeral ligament


2. Acromioclavicular (AC) ligaments
3. Coracoclavicular ligaments
4. Coracoacromial ligament

PREFERRED RESPONSE: 3

DISCUSSION:
The radiograph shows an extra-articular distal clavicle :fracture lateral to the clavicular attachment point of
the coracoclavicular ligaments (conoid and trapezoid). However, unlike a scenario featuring a typical Neer
type I :fracture, the interval between coracoid and clavicle is clearly widened and there is marked :fracture
displacement. It is clear that the coracoclavicular ligaments must also be tom. The inferior glenohumeral
ligament is important to glenohumeral joint stability, but has no effect on the relationship between
clavicle and scapula. The AC ligaments are thickenings of the AC joint capsule. They have been shown
to be responsible for 90% of anteroposterior stability of the AC joint. The coracoclavicular ligaments are
responsible for 77% of stability for superior translation (as in this case). The coracoacrornial ligarnent
connects 2 parts of the scapula (coracoids and acromion) and is part of the arch that supports the rotator
cuff.

RECOMMENDED READINGS
NEER CS 2nd. Fracture of the distal clavicle with detachment of the coracoclavicular ligaments in adults.
J Trauma. 1963 Mar;3:99-110. PubMed PMID: 13937900.

URIST MR. COMPLETE DISLOCATION OF THE ACROMIOCLAVICULAR JOINT. J Bone Joint


SurgArn. 1963 Dec;45:1750-3. PubMed PMID: 14083156.

Acus RW 3rd, Bell RH, Fisher DL. Proximal clavicle excision: an analysis ofresults. J Shoulder Elbow
Surg. 1995 May-Jun;4(3):182-7. PubMedPMID: 7552675.

© 2014 American Academy of Orthopaedic Surgeons 2014 Shoulder and Elbow Self-Assessment Examination
26 • American Academy of Orthopaedic Surgeons

Figure 18

Question 18
A 36-year-old right-hand-dominant man fell from his motorcycle and sustained the acute right upper
extremity injury seen in Figure 18. At surgery, an open reduction and internal :fixation of the ulna is
performed along with attempted open reduction of the radiocapitellar joint. However, the radial head is
slightly subluxed in fiexion and redislocates with elbow extension below 90 degrees. What is the most
appropriate treatment at this time?

1. Radial head resection


2. Casting in 90 degrees of flexion for 3 weeks, followed by reassessment of elbow stability
3. Repair of the annular ligament
4. Revision open reduction and internal fixation of the ulnar fracture

PREFERRED RESPONSE: 4

DISCUSSION
This case is a variant of a type I Monteggia fracture according to the Bado classification with a segmental
ulna fracture. In some cases, the radial head subluxation can be subtle, and missing this would lead to a
poor outcome. In this case, the anterior radial head dislocation is obvious, but the segmental nature of the
ulna fracture makes anatomic reduction difficult. The radial head usually spontaneously reduces once the
ulna is anatomically reduced, and no surgical treatment to the lateral side is required. When this is not the
case, a lateral approach and incision of the annular ligament may be required for reduction. If an open
reduction of the radial head is unsuccessful, the problem is almost always residual malalignment of the
ulna. Therefore, casting and annular ligament repair will not improve reduction. A radial head resection
would eliminate the nonconcentric contact between radial head and capitellum, but would not be an
appropriate treatment for this young patient who has an acute, correctable fracture deformity.

© 2014 American Academy of Orthopaedic Surgeons 2014 Shoulder and Elbow Self-AssessmentExamination
2014 Shoulder and Elbow Self-Assessment Examination Answer Book• 27

RECOMMENDED READINGS
Bado JL. The Monteggia lesion. Clin Orthop Relat Res. 1967 Jan-Feb;S0:71-86. PubMed PMID: 6029027.

Ring D, Jupiter JB, Waters PM. Monteggia fractures in children and adults. J Am Acad Orthop Surg. 1998
Jul-Aug;6(4):215-24. Review. PubMed PMID: 9682084.

Figure 19

Question 19
Figure 19 is the radiograph of a 45-year-old right-hand-dominant man who has had a 2-day history of right
shoulder pain, weakness, and a deformity involving the clavicle region after a fall from a scaffold during
work activities. He was previously evaluated by his primary care physician and another orthopaedist. He
has sought a second opinion regarding his treatment options. What is the most appropriate treatment for
his injury?

1. Sling immobilization with continued observation


2. Open reduction and plate fixation
3. Open reduction, plate fixation, and application of an external bone stimulator
4. External fixation

PREFERRED RESPONSE: 2

DISCUSSION
Midshaft clavicle fractures are relatively common and account for 3% to 10% of all adult fractures.
Historical studies regarding nonsurgical treatment of displaced clavicle fractures indicated a low nonunion
rate approaching 1%. Contemporary studies, however, suggest that the nonunion rate is much higher,
reaching 15% to 20% with corresponding loss of shoulder strength and residual deficits at 6 months from
date of injury. A recent meta-analysis by McKee and associates comparing nonsurgical treatment with
a sling and surgical treatment with plate fixation concluded that initial fixation of displaced midshaft
clavicle fractures demonstrated a positive effect on pain reduction at 3 weeks and improved Disabilities of
the Arm, Shoulder, and Hand (DASH) scores at 6 weeks and 3 months after surgery. The choice of fixation
remains a matter of debate because both plate and pin fixation have been used to achieve clavicle union.
Intramedullary fixation of clavicle fractures historically has demonstrated an increased risk for pin tract
infections and hardware removal attributable to local irritation from the implant. External bone stimulator
use has not proven beneficial in effecting a reduction of nonunion rates. The most common complications
noted with surgical intervention included local hardware irritation and wound infection.
IC 2014 American Academy of Orthopaedic Surgeons 2014 Shoulder and Elbow Self-Assessment Examination
28 • American Academy of Orthopaedic Surgeons

RECOMMENDED READINGS
McKee RC, Whelan DB, Schemitsch EH, McKee MD. Operative versus nonoperative care of displaced
midshaft clavicular fractures: a meta-analysis ofrandomized clinical trials. J Bone Joint SurgAm. 2012
Apr 18;94(8):675-84. Review. PubMed PMID: 22419410.

Canadian Orthopaedic Trauma Society. Nonoperative treatment compared with plate fixation of displaced
midshaft clavicular fractures. A multicenter, randomized clinical trial. J Bone Joint Surg Am. 2007
Jan;89(1):1-10. PubMed PMID: 17200303.

Judd DB, Pallis MP, Smith E, Bottoni CR. Acute operative stabilization versus nonoperative management
of clavicle fractures. Am J Orthop (Belle Mead NJ). 2009 Jul;38(7):341-5. PubMed PMID: 19714275.

Question 20
A 55-year-old woman develops posttraumatic arthritis in the elbow following a distal humerus fracture.
What is the most likely mid-term (5-10 years after surgery) complication following semiconstrained total
elbow arthroplasty (TEA)?

1. Bushing wear
2. Infection
3. Aseptic component loosening
4. Component fracture

PREFERRED RESPONSE: 1

DISCUSSION
TEA has been described for posttraumatic arthritis of the elbow and typically involves a young patient
population with multiple previous operations on the affected elbow. Morrey and Schneeberger found
aseptic component loosening to be uncommon (fewer than 10% of patients), and usually occurs more
than 10 years after surgery. Prosthetic fracture, usually of the ulnar component, is also a late-term finding.
Infection is the most common mode of early failure, but usually occurs within the first 5 years and has
an overall rate of approximately 5%. Bushing wear has been reported as the most common cause of
mechanical TEA failure in this population at intermediate-term followup.

RECOMMENDED READINGS
Schneeberger AG, Adams R, Morrey BF. Semiconstrained total elbow replacement for the treatment of
post-traumatic osteoarthrosis. J Bone Joint SurgAm. 1997 Aug;79(8):1211-22. PubMed PMID: 9278082.

Morrey BF, Schneeberger AG. Total elbow arthroplasty for posttraumatic arthrosis. Instr Course Leet.
2009;58:495-504. PubMed PMID: 19385558.

Throckmorton T, Zarkadas P, Sanchez-Sotelo J, Morrey B. Failure patterns after linked semiconstrained


total elbow arthroplasty for posttraumatic arthritis. J Bone Joint SurgAm. 2010 Jun;92(6):1432-41. doi:
10.2106/JBJS.I.00145. PubMed PMID: 20516319.

© 2014 American Academy of Orthopaedic Surgeons 2014 Shoulder and Elbow Self-Assessment Examination
2014 Shoulder and Elbow Self-Assessment Examination Answer Book • 29

CLINICAL SITUATION FOR QUESTIONS 21 AND 22


A 21-year-old rugby player has had the sensation of shoulder instability while making tackles for 3 years.
Two years ago, he had an arthroscopic Bankart repair and capsulorrhaphy that used 3 suture anchors after
dislocating his shoulder while making a tackle. This procedure required an emergency department sedated
reduction. After this dislocation, he had paresthesias in his arm and a sense of weakness. His numbness
eventually resolved. He did well after surgery until 2 weeks ago, when he again felt his shoulder dislocate
while tackling and had an emergency department reduction.

Question 21
What caused his recurrent instability?

1. The use of suture anchors in his repair


2. The physical therapy program after surgery
3. His age at the time of first surgery
4. His activity levels after surgery

PREFERRED RESPONSE: 4

Question 22
Numbness after his first dislocation was related to

1. intrasurgical traction on the musculocutaneous nerve.


2. residual interscalene blockade.
3. ulnar neuropathy after sling use.
4. sensory axillary nerve palsy from his dislocation.

PREFERRED RESPONSE: 4

DISCUSSION FOR QUESTIONS 21 AND 22


Several studies have demonstrated the equivalency of arthroscopic Bankart repair to open surgery, but
most studies have also demonstrated increased recurrence rates in patients who participate in collision
sports such as rugby. Revision surgery with arthroscopy is unlikely to be durable, and in patients who have
large glenoid defects, a coracoid process transfer (Latarjet or Bristow) is the preferred surgical treatment.
Arthroplasty would not be indicated in a young and active patient.

11;) 2014 American Academy of Orthopaedic Surgeons 2014 Shoulder and Elbow Self-Assessment Examination
30 • American Academy of Orthopaedic Surgeons

RECOMMENDED READINGS FOR QUESTIONS 21 AND 22


Neyton L, Young A, Dawidziak B, Visona E, Hager JP, Fournier Y, Walch G. Surgical treatment of
anterior instability in rugby union players: clinical and radiographic results of the Latarjet-Patte procedure
with minimum 5-year follow-up. J Shoulder Elbow Surg. 2012 Dec;21(12):l 721-7. doi: 10.1016/j.
jse.2012.01.023. Epub 2012 May 5. PMID 22565042.

Robinson CM, Shur N, Sharpe T, Ray A, Murray IR. Injuries associated with traumatic anterior
glenohumeral dislocations. J Bone Joint SurgAm. 2012 Jan 4;94(1):18-26. PMID: 22218378.

END OF SERIES

© 2014 American Academy of Orthopaedic Surgeons 2014 Shoulder and Elbow Self-Assessment Examination
2014 Shoulder and Elbow Self-Assessment Examination Answer Book• 31

Figure 23a Figure 23b

Figure 23c Figure 23d

Question 23
Figures 23a through 23d are the radiographs and Jv1R1 scans of a 30-year-old otherwise healthy man who
sustained an anterior right shoulder dislocation while playing baseball. He requires a closed reduction
under sedation at a local emergency department. He is placed into a shoulder immobilizer and referred to
your office for further treatment. Upon inquiry, the patient states that he previously dislocated the shoulder
twice within the last year while playing basketball. He demonstrates positive apprehension and speed tests.
What is the most appropriate next treatment step?

1. Brief period of immobilization with initiation of therapy


2. Arthroscopic labral repair
3. Open capsular shift
4. Coracoid transfer

PREFERRED RESPONSE: 2

© 2014 American Academy of Orthopaedic Surgeons 2014 Shoulder and Elbow Self-Assessment Examination
32 • American Academy of Orthopaedic Surgeons

DISCUSSION
By history, this patient has recurrent anterior instability. His radiographs do not indicate significant
deficiency of the anterior glenoid, which would potentially require augmentation with a coracoid transfer
to restore stability to the glenohumeral joint. An MRI scan reveals a displaced anteroinferior labral
tear (Bankart lesion) with extension into the biceps labral complex. An open capsular shift can address
capsular redundancy, but an arthroscopic procedure will allow for examination of concomitant pathology
and the ability to address the capsulolabral tear with reduced morbidity. To minimize this patient's
redislocation risk with sports activities, an arthroscopic capsulolabral repair involving the anteroinferior
and superior labrum is the most appropriate solution. Hantes and associates demonstrated that clinical
outcomes are similar at 2-year followup in patients with combined anteroinferior and superior labral tears
vs patients with isolated Bankart lesions when treated by arthroscopic means.

RECO:MMENDED READINGS
Carreira DS, Mazzocca AD, Oryhon J, Brown FM, Hayden JK, Romeo AA. Aprospective outcome
evaluation of arthroscopic Bankart repairs: minimum 2-year follow-up. Am J Sports Med. 2006
May;34(5):771-7. PubMed PMID: 16627629.

Taylor DC, Arciero RA. Pathologic changes associated with shoulder dislocations. Arthroscopic and
physical examination findings in first-time, traumatic anterior dislocations. Am J Sports Med. 1997 May-
Jun;25(3):306-l1. PubMed PMID: 9167808.

Hantes ME, Venouziou AI, Liantsis AK, Dailiana ZH, Malizos KN. Arthroscopic repair for chronic
anterior shoulder instability: a comparative study between patients with Bankart lesions and patients with
combined Bankart and superior labral anterior posterior lesions. Am J Sports Med. 2009 Jun;37(6):1093-8.
Epub 2009 Mar 13. PubMed PMID: 19286910.

CLINICAL SITUATION FOR QUESTIONS 24 THROUGH 26


A 16-year-old competitive skier sustained an anterior shoulder dislocation. Her shoulder was reduced in
the emergency department and placed in a sling and swathe. She follows up with you 2 weeks later.

Question 24
You should counsel this patient and family

1. to have immediate surgery so that she may finish the ski season.
2. that external rotation bracing now will prevent recurrence.
3. that even with a large bone defect (>20%), arthroscopic surgery is successful.
4. that 2 weeks of immobilization followed by therapy may allow her to return to finish the
season.

PREFERRED RESPONSE: 4

© 2014 American Academy of Orthopaedic Surgeons 2014 Shoulder and Elbow Self-Assessment Examination
2014 Shoulder and Elbow Self-Assessment Examination Answer Book• 33

Question 25
The family opts for nonsurgical treatment with therapy and a brace to finish her season. Because
instability symptoms continue, an MRI scan is obtained and reveals a Bankart lesion. You
recommend

1. thermal capsulorrhaphy.
2. arthroscopic Bankart repair.
3. arthroscopic Latarjet.
4. open Magnuson-Stack.

PREFERRED RESPONSE: 2

Figure 24

Question 26
The patient underwent an uneventful arthroscopic repair and did well until 1 year later when she crashed
during a race. She tore her anterior cruciate ligament (ACL) and underwent reconstruction. Followup after
her successful ACL reconstruction reveals complaints of new shoulder pain and posterior instability from
using crutches after her ACL surgery. A new MRI scan is shown in Figure 24. What factors are most likely
associated with this patient's recurrence?

1. Gender
2. Age
3. Sport
4. Bone loss

PREFERRED RESPONSE: 2

© 2014 American Academy of Orthopaedic Surgeons 2014 Shoulder and Elbow Self-Assessment Examination
34 • American Academy of Orthopaedic Surgeons

DISCUSSION FOR QUESTIONS 24 THROUGH 26


Nonsurgical treatment for a first-time traumatic dislocation may not be effective in a young, athletic
patient; moreover, a Bankart lesion may be present in 70% of all traumatic subluxations. A period of
immobilization in external rotation may decrease the risk for recurrence, but only if the patient is placed
in a brace immediately and complies with the treatment. Return to play may be possible after 2 weeks
of immobilization, provided the patient undergoes appropriate range of motion, cuff strengthening, and
scapular stabilization exercises. Among athletes in 1 study, 80% were able to finish the season, but 40%
experienced episodes of subluxation or recurrent dislocation prior to the end of the season.

The rate of recurrence after arthroscopic stabilization should be lower than 10% and may be equivalent to
open repair for most patients. Bone loss remains the primary indication for open procedures, as coracoid
transfers or other bone-grafting operations may be needed to reconstruct the glenoid if more than 20%
of its surface area is missing. Furthermore, collision athletes may fare better with open surgery than
arthroscopic options. Bone loss remains the most significant factor for recurrence across many studies.
Glenoid bone loss may be present in 20% of primary dislocations and 70% of recurrent dislocations.
Age younger than 30 has a high correlation with recurrence. Although men may be almost twice as
likely as women to have a recurrent dislocation, age seems to be the most reliable patient-related risk
factor for recurrence. Thermal capsulorrhaphy has not proven to be effective and carries a high risk for
complication. More recent studies have found equal recurrence rates between open and arthroscopic
Bankart repair, with a greater loss of motion in patients who underwent open repair. Longitudinal studies
have demonstrated that 40% to 50% are likely to develop arthritis after a shoulder dislocation; however,
recurrent dislocation seems to be the most important factor for early development of arthritis, while age
younger than 25 may be protective. Postcapsulorrhaphy arthropathy may be more associated with open
repairs or those that severely limit external rotation.

© 2014 American Academy of Orthopaedic Surgeons 2014 Shoulder and Elbow Self-Assessment Examination
2014 Shoulder and Elbow Self-Assessment Examination Answer Book• 35

RECOMMENDED READINGS FOR QUESTIONS 24 THROUGH 26


DeAngelis NA, Busconi BD, Mozzocca AD, Arciero RA. Recurrent anterior shoulder instability. In:
Galatz LM, ed. Orthopaedic Knowledge Update Shoulder and Elbow 3. Rosemont, IL: American
Academy of Orthopaedic Surgeons; 2008:93-102.

Ahmed I, Ashton F, Robinson CM. Arthroscopic Ban.kart repair and capsular shift for recurrent anterior
shoulder instability: functional outcomes and identification of risk factors for recurrence. J Bone Joint
SurgAm. 2012 Jul 18;94(14):1308-15. doi: 10.2106/JBJS.J.01983. PubMed PMID: 22810402.

Boileau P, Villalba M, Hery JY, Balg F, Ahrens P, Neyton L. Risk factors for recurrence of shoulder
instability after arthroscopic Ban.kart repair. J Bone Joint Surg Am. 2006 Aug;88(8):1755-63. PubMed
PMID: 16882898.

Paterson WH, Throckmorton TW, Koester M, Azar FM, Kuhn JE. Position and duration of immobilization
after primary anterior shoulder dislocation: a systematic review and meta-analysis of the literature. J
Bone Joint SurgAm. 2010 Dec 15;92(18):2924-33. doi: 10.2106/JBJS.J.00631. Review. PubMed PMID:
21159993.

Boileau P, Fourati E, Bicknell R. Neer modification of open Bankart procedure: what are the rates of
recurrent instability, functional outcome, and arthritis? Clin Orthop Relat Res. 2012 Sep;470(9):2554-60.
doi: 10.1007/s11999-012-2296-5. Epub 2012 Mar 1. PubMed PMID: 22395874.

Hovelius L, Sandstrom B, Saebo M. One hundred eighteen Bristow-Latarjet repairs for recurrent anterior
dislocation of the shoulder prospectively followed for fifteen years: study II-the evolution of dislocation
arthropathy. J Shoulder Elbow Surg. 2006 May-Jun;15(3):279-89. PubMed PMID: 16679226.

Hovelius L, Saeboe M. Neer Award 2008: Arthropathy after primary anterior shoulder dislocation--223
shoulders prospectively followed up for twenty-five years. J Shoulder Elbow Surg. 2009 May-
Jun;18(3):339-47. doi: 10.1016/j.jse.2008.11.004. Epub 2009 Feb 28. PubMed PMID: 19254851.

END OF SERIES

© 2014 American Academy of Orthopaedic Surgeons 2014 Shoulder and Elbow Self-Assessment Examination
36 • American Academy of Orthopaedic Surgeons

Figure 27

Question 27
Figure 27 is the MRI scan of a 63-year-old man who has experienced 3 years of shoulder pain. He has had
2 fluoroscopically guided corticosteroid injections that provided him with temporary but significant relief.
His primary care physician ordered an MRI scan because of his ongoing shoulder issues. His examination
shows significant loss of motion in all planes but good motor strength. The best treatment at this point
would include

1. hyaluronic acid injection.


2. physical therapy.
3. reverse total shoulder arthroplasty.
4. anatomic total shoulder arthroplasty.

PREFERRED RESPONSE: 4

DISCUSSION
Glenohumeral arthritis in this age group is best treated with total shoulder arthroplasty. It provides more
durable and cost-effective relief than hemiarthroplasty. Hyaluronic acid has been demonstrated in some
studies to be effective at improving pain in the short term, but is not approved by the U.S. Food and Drug
Administration for use in the shoulder. Reverse total shoulder replacement is indicated in patients who
have an irreparable rotator cuff tear. This image clearly shows the supraspinatus is intact. Physical therapy
has not proven beneficial in the setting of end-stage glenohumeral arthritis.

© 2014 American Academy of Ortbopaedic Surgeons 2014 Shoulder and Elbow Self-Assessment Examination
2014 Shoulder and Elbow Self-Assessment Examination Answer Book• 37

RECOMMENDED READINGS
Izquierdo R, Voloshin I, Edwards S, Freehill MQ, Stanwood W, Wiater JM, Watters WC 3rd, Goldberg
MJ, Keith M, Turkelson CM, Wies JL, Anderson S, Boyer K, Raymond L, Sluka P; Treatment of
glenohumeral osteoarthritis. American Academy of Orthopedic Surgeons. J Am Acad Orthop Surg. 2010
Jun;18(6):375-82. PMID: 20511443.

Mather RC 3rd, Watters TS, Orlando LA, Bolognesi MP, Moorman CT 3rd. Cost effectiveness analysis of
hemiarthroplasty and total shoulder arthroplasty. J Shoulder Elbow Surg. 2010 Apr;19(3):325-34. PMID:
20303459.

Edwards TB, Kadakia NR, BoulahiaA, Kempf JF, Boileau P, Nemoz C, Walch G. A comparison of
hemiarthroplasty and total shoulder arthroplasty in the treatment of primary glenohumeral osteoarthritis:
results of a multicenter study. J Shoulder Elbow Surg. 2003 May-Jun;12(3):207-13. PMID: 12851570.

Gartsman GM, Roddey TS, Hammerman SM. Shoulder arthroplasty with or without resurfacing of
the glenoid in patients who have osteoarthritis. J Bone Joint Surg Am. 2000 Jan;82(1):26-34. PMID:
10653081.

Figure 28

CLINICAL SITUATION FOR QUESTIONS 28 AND 29


Figure 28 is the radiograph of a 39-year-old man who had a syncopal episode and fall. After being cleared
by the emergency department, he is referred to your office for left shoulder pain and loss of external
rotation.

© 2014 American Academy of Orthopaedic Surgeons 2014 Shoulder and Elbow Self-Assessment Examination
38 • American Academy of Orthopaedic Surgeons

Question 28
What is the most likely diagnosis?

1. Rotator cuff tear


2. Adhesive capsulitis
3. Brachia! plexus injury
4. Posterior shoulder dislocation

PREFERRED RESPONSE: 4

Question 29
What is the best next step?

1. Physical therapy
2. CTscan
3. Arthroscopic capsular release
4. Arthroscopic Bankart repair

PREFERRED RESPONSE: 2

DISCUSSION FOR QUESTIONS 28 AND 29


The patient has a posterior glenohumeral dislocation, as evidenced by the overlap on the initial radiograph.
While posterior dislocations are rare, they can be overlooked. A CT scan will accurately show the lesion
prior to a trip to the operating room. If a simple closed reduction is performed acutely and the arm is
stable after the reduction, no further intervention may be needed and treatment can be successful with a
2- week period of immobilization for defects involving less than 30% of the humeral head. However, in
this scenario, open reduction is likely and stabilization may require a modified McLaughlin procedure or
other intervention to fill in the humeral defect. Younger male patients, those with a large humeral head
defect, and those with seizure disorder may be at highest risk for recurrence. For treatment of chronic
posterior dislocations, it may be necessary to perform shoulder arthroplasty to restore stability. Stiffness is
attributable to articular incongruity; therefore, physical therapy and capsular release are inappropriate.

© 2014 American Academy of Orthopaedic Surgeons 2014 Shoulder and Elbow Self-Assessment Examination
2014 Shoulder and Elbow Self-Assessment Examination Answer Book• 39

RECOMMENDED READINGS FOR QUESTIONS 28 AND 29


Duralde XA, Fogle EF. The success of closed reduction in acute locked posterior fracture-dislocations
of the shoulder. J Shoulder Elbow Surg. 2006 Nov-Dec;15(6):701-6. Epub 2006 Oct 19. PubMed PMID:
17055305.

Robinson CM, Aderinto J. Posterior shoulder dislocations and fracture-dislocations. J Bone Joint Surg
Am. 2005 Mar;87(3):639-50. Review. PubMed PMID: 15741636.

Robinson CM, Seah M, Akhtar MA. The epidemiology, risk of recurrence, and functional outcome after
an acute traumatic posterior dislocation of the shoulder. J Bone Joint SurgAm. 2011 Sep 7;93(17):1605-
13. doi: 10.2106/JBJS.J.00973. PubMed PMID: 21915575.

Sperling JW, Pring M, Antuna SA, Cofield RH. Shoulder arthroplasty for locked posterior dislocation of
the shoulder. J Shoulder Elbow Surg. 2004 Sep-Oct;13(5):522-7. PubMed PMID: 15383808.

END OF SERIES

Question 30
A 40-year-old right-hand-dominant construction worker has a 3-month history of right shoulder weakness
secondary to a fall from a ladder at work. He underwent nonsurgical treatment with anti-inflammatory
medication, cortisone injections, and therapy, with minimal relief of his symptoms. A subsequent l\1RI
scan indicates a 1-cm full-thickness supraspinatus tendon tear. He has been referred to your clinic for
discussion of surgical intervention. The patient's nurse case manager is concerned that he may not be able
to return to his preinjury level of activity at work, even with surgical intervention. You tell the nurse case
manager that, on average, the patient will

1. be at increased risk for infection compared to patients without a Worker's Compensation


claim.
2. have significant functional improvement after rotator cuff repair that is less robust than that of
patients without a Worker's Compensation claim.
3. have pain relief that is equivalent to that of patients without a Worker's Compensation claim.
4. return to work without restrictions within a 3-month time frame.

PREFERRED RESPONSE: 2

© 2014 American Academy of Orthopaedic Surgeons 2014 Shoulder and Elbow Self-Assessment Examination
40 • American Academy of Orthopaedic Surgeons

DISCUSSION
Many factors have been associated with less-than-favorable outcomes following rotator cuff repair such as
tear size, age at time of intervention, gender, fatty degeneration of rotator cuff musculature, and Worker's
Compensation status. Henn and associates performed a prospective study regarding rotator cuff repairs
in a cohort of 125 patients to assess the factors that may affect outcome as measured with the Simple
Shoulder Test (SST), Disabilities of the Arm, Shoulder, and Hand (DASH), Short Fonn-36 (SF-36), and
Visual Analog Pain Scale (VAS). When confounding factors were controlled, Worker's Compensation
status was an independent predictor of poorer DASH scores. With the use of historical controls, Bhatia and
associates concluded that the vast majority (89%) of workers who underwent an arthroscopic rotator cuff
repair returned to their preoperative level of work at a mean time of7.6 months. There was a trend toward
decreased return to full duty with increased work demands before surgery (light, medium, and heavy
duty), but this result did reach statistical significance. Alcohol consumption (more than 6 drinks per week)
was the only factor to demonstrate an association with postoperative restricted work duty and increased
rotator cuff repair failure. There is no evidence to support increased infection rates for rotator cuff repair in
Worker's Compensation patients.

RECOMMENDED READINGS
Bhatia S, Piasecki DP, Nho SJ, Romeo AA, Cole BJ, Nicholson GP, Boniquit N, Verma NN. Early
return to work in workers' compensation patients after arthroscopic full-thickness rotator cuff repair.
Arthroscopy. 2010 Aug;26(8):1027-34. Epub 2010 Jun 3. PubMed PMID: 20678699.

Henn RF 3rd, Kang L, Tashjian RZ, Green A. Patients with workers' compensation claims have worse
outcomes after rotator cuff repair. J Bone Joint Surg Am. 2008 Oct;90(10):2105-13. PubMed PMID:
18829907.

Question 31
A 75-year-old man sustains an anterior dislocation of his reverse total shoulder arthroplasty. What activity
places the arm in the position most commonly associated with reverse total shoulder dislocation?

1. Scratching the opposite shoulder


2. Pushing off an ipsilateral chair armrest to assist in standing up
3. Tying shoelaces on the contralateral foot
4. Brushing hair

PREFERRED RESPONSE: 2

DISCUSSION
Proper soft-tissue tension is critical to prevent instability of a reverse total shoulder implanted with
the deltopectoral approach; dislocation of the prosthesis is exceedingly rare if the superior approach is
employed. The arm position implicated in reverse total shoulder instability is extension, adduction, and
internal rotation, such as pushing out of a chair. The other positions described do not involve extension of
the shoulder.

© 2014 American Academy of Orthopaedic Surgeons 2014 Shoulder and Elbow Self-Assessment Examination
2014 Shoulder and Elbow Self-Assessment Examination Answer Book• 41

RECOMMENDED READINGS
Gerber C, Pennington SD, Nyffeler RW. Reverse total shoulder arthroplasty. J Am Acad Orthop Surg.
2009 May;17(5):284-95. Review. PubMed PMID: 19411640.

Cheung E, Willis M, Walker M, Clark R, Frankle MA. Complications in reverse total shoulder
arthroplasty. J Am Acad Orthop Surg. 2011 Jul;19(7):439-49. Review. PubMed PMID: 21724923.

Walch G, Wall B, Mottier F: Complication and revision of the reverse prosthesis: A multicenter study
of 457 cases. In: Walch G, Boileau P, Mole P, Favard L, Levigne C, Sirveaux F, eds. Reverse Shoulder
Arthroplasty: Clinical Results, Complications, Revision. Montpellier, France: Sauramps Medical;
2006:335-352.

Question 32
When performing an ulnar nerve decompression at the elbow, the surgeon must be aware of the

1. median nerve as it crosses the surgical field 6 cm proximal to the medial epicondyle.
2. medial antebrachial cutaneous nerve as it crosses the field 3 cm distal to the medial
epicondyle.
3. anterior antebrachial cutaneous nerve as it crosses the field at the medial epicondyle.
4. posterior antebrachial cutaneous nerve that crosses the field 2 cm distal to the medial
epicondyle.

PREFERRED RESPONSE: 2

DISCUSSION
The medial antebrachial cutaneous and medial brachial cutaneous are nerves that can be injured during
ulnar nerve decompression at the elbow. The medial antebrachial cutaneous nerve crosses the surgical
field at an average of 3.1 cm distal to the medial epicondyle. The medial brachia! cutaneous nerve crosses
the field 7 cm proximal to the medial epicondyle and arborizes into 2 to 3 terminal branches. Because the
surgical approach involves dissection on the medial side, the posterior antebrachial cutaneous nerve is
distant from the exposure. Although the median nerve potentially can be located in the deep dissection of a
submuscular transposition, it is considered distant to an in situ decompression.

RECOMMENDED READINGS
Lowe JB 3rd, Maggi SP, Mackinnon SE. The position of crossing branches of the medial antebrachial
cutaneous nerve during cubital tunnel surgery in humans. Plast Reconstr Surg. 2004 Sep 1;114(3):692-6.
PubMed PMID: 15318047.

Chowdhry S, Elston JB, Lefkowitz T, Wilhelmi BJ. Avoiding the medial brachia! cutaneous nerve in
brachioplasty: an anatomical study. Eplasty. 2010 Jan 29;10:el6. PubMed PMID: 20165546.

© 2014 American Academy of Orthopaedic Surgeons 2014 Shoulder and Elbow Self-Assessment Examination
42 • American Academy of Orthopaedic Surgeons

Figure 33

Question 33
Figure 33 is the radiograph of a 27-year-old bicyclist who crashes. He has an isolated and closed injury.
He is neurovascularly intact in the upper extremity. The lateral fragment is displaced inferiorly by

1. gravity.
2. the trapezius.
3. the biceps.
4. the pectoralis minor.

PREFERRED RESPONSE: 1

DISCUSSION
Open reduction and internal fixation with a plate and screw construct have been demonstrated to reduce
nonunion rate and improve outcomes compared to sling immobilization for displaced clavicle fractures.
Neurovascular injury and infection risk increase, however, with surgery. In the upright position, the weight
of the extremity inferiorly displaces the lateral segment.

RECOJ\1MENDED READINGS
Altamimi SA, McKee MD; Canadian Orthopaedic Trauma Society. Nonoperative treatment compared
with plate fixation of displaced midshaft clavicular fractures. Surgical technique. J Bone Joint Surg Am.
2008 Mar;90 Suppl 2 Pt 1:1-8. PMID: 18310682.

Canadian Orthopaedic Trauma Society. Nonoperative treatment compared with plate fixation of displaced
midshaft clavicular fractures. A multicenter, randomized clinical trial. J Bone Joint Surg Am. 2007
Jan;89(1):l-10. PMID: 17200303.

Darouiche RO. Treatment of infections associated with surgical implants. N Engl J Med. 2004 Apr
1;350(14):1422-9. Review. PubMed PMID: 15070792.

ll:l 2014 American Academy of Orthopaedic Surgeons 2014 Shoulder and Elbow Self-Assessment Examination
2014 Shoulder and Elbow Self-Assessment Examination Answer Book• 43

Figure 34a Figure 34b

Question 34
Placement of the most distal interlocking screw seen in the Figures 34a and 34b radiographs most likely
resulted in what motor weakness?

1. Elbow :flexion
2. Thumb interphalangeal (IP) extension
3. Index proximal IP fl.exion
4. Index metacarpophalangeal (MCP) abduction

PREFERRED RESPONSE: 3

DISCUSSION
The most distal locking screw in this intramedullary nail construct was placed from anterior to posterior,
passing through the distal portion of the biceps and brachialis muscle bellies. The median nerve, along
with the brachia! artery, is at risk as it lies between these 2 muscles. Malrotation of the nail, producing
a more anteromedial starting point for the screw, can lead to a path that intersects with the nerve. Blunt
dissection and soft-tissue protection is warranted with this screw placement. Median nerve injury would
affect innervations of the flexor digitorum super:ficialis and profundus to the index :finger (among other
motors). Although the dissection violates the muscle belly of these 2 elbow :flexors, measurable weakness
is not typically seen. The radial nerve has already provided function to triceps (elbow extension) proximal
to this level and lies sufficiently lateral to be more of a concern with a lateral screw placement (thumb
IP extension). The ulnar nerve (index MCP abduction) is further medial at this level and would be at
considerably lower risk than the median.

RECOJV0.\1ENDED READINGS
Morrey BF. Anatomy of the elbow joint. In: Morrey BF, Sanchez-Sotelo J, eds. The Elbow and Its
Disorders. Philadelphia, PA: WB Saunders; 2009:11-38.

Bohsali Kl, Wirth MA. Fractures of the proximal humerus. In: Rockwood CA, Matsen FA, Wirth MA,
Lippitt SB, eds. The Shoulder. Philadelphia, PA: WB Saunders; 2009:315-319.

© 2014 American Academy of Orthopaedic Surgeons 2014 Shoulder and Elbow Self-Assessment Examination
44 • American Academy of Orthopaedic Surgeons

Question 35
One week after closed reduction of a primary anterior shoulder dislocation, a 25-year-old athlete should be
counseled that

1. recurrence rate is reduced with 4 weeks of immobilization instead of 2 weeks of


immobilization.
2. age at the time of injury is the most consistent risk factor for recurrent instability.
3. a majority of patients in this age group will elect to have surgery for recurrent instability.
4. after an in-season return to sports, his likelihood of choosing surgery after the season is 25%.

PREFERRED RESPONSE: 2

DISCUSSION
In a study by Sachs and associates, age younger than 25 years at the time of presentation was found to be
the strongest predictor ofrecurrent instability. In this age group (20- to 29-year-olds), only 14% elected to
proceed with surgery. After an in-season return to sports, about 50% of patients in this same study chose to
proceed with surgery after completing the season. Immobilization in a sling for longer than 2 weeks has no
effect on future instability.

RECO:MMENDED READINGS
Sachs RA, Lin D, Stone l\1L, Paxton E, Kuney M. Can the need for future surgery for acute traumatic
anterior shoulder dislocation be predicted? J Bone Joint Surg Am. 2007 Aug;89(8):1665-74. PubMed
PMID: 17671003.

Paterson WH, Throckmorton TW, Koester M, Azar FM, Kuhn JE. Position and duration of immobilization
after primary anterior shoulder dislocation: a systematic review and meta-analysis of the literature. J
Bone Joint SurgAm. 2010 Dec 15;92(18):2924-33. doi: 10.2106/JBJS.J.0063l. Review. PubMed PMID:
21159993.

CLINICAL SITUATION FOR QUESTIONS 36 THROUGH 39


A 65-year-old man experienced 6 years of worsening shoulder pain. Examination demonstrates stiffness
and crepitus with range of motion, but full rotator cuff strength in all planes. Radiographs show advanced
shoulder osteoarthritis, and an MRI scan ordered by the patient's primary care physician shows an intact
rotator cuff.

© 2014 American Academy of Orthopaedic Surgeons 2014 Shoulder and Elbow Self-Assessment Examination
2014 Shoulder and Elbow Self-Assessment Examination Answer Book • 45

Question 36
What is the most likely glenoid wear pattern seen in glenohumeral osteoarthritis with an external rotation
deficit?

1. Posterior wear
2. Anterior wear
3. Central wear
4. Superior wear

PREFERRED RESPONSE: 1

Question 37
What surgical treatment is most likely to result in long-term pain relief and functional improvement?

1. Hemiarthroplasty
2. Hemiarthroplasty with meniscal interposition
3. Total shoulder arthroplasty (TSA)
4. Reverse TSA

PREFERRED RESPONSE: 3

Question 38
What risk factor is most predictive of deep infection following TSA?

1. Posttraumatic arthritis
2. Male gender
3. Body mass index higher than 30
4. Diabetes

PREFERRED RESPONSE: 2

© 2014 American Academy of Orthopaedic Surgeons 2014 Shoulder and Elbow Self-Assessment Examination
46 • American Academy of Orthopaedic Surgeons

Question 39
At what point of glenoid retroversion is there risk for component perforation of the glenoid vault with
traditional high side reaming and standard component implantation?

1. 5 degrees
2. 10 degrees
3. 15 degrees
4. 20 degrees

PREFERRED RESPONSE: 4

DISCUSSION FOR QUESTIONS 36 THROUGH 39


Posterior glenoid wear is the most common pattern seen in typical glenohumeral arthritis. Central wear
can also be seen, but it is less common and anterior wear is least common. TSA is the gold standard for
surgical treatment of glenohumeral osteoarthritis. Multiple comparative studies between hemiarthroplasty
and total shoulder arthroplasty demonstrate advantages of TSA regarding pain relief and most functional
parameters. Shoulder hemiarthroplasty with meniscal interposition has been described for young patients
with glenohumeral arthritis, but outcomes at intermediate-term followup have been inferior to those of
TSA. Reverse TSA is contraindicated in patients with an intact rotator cuff. Proprionobacterium acnes
and Staphylococcus species are the most common pathogens causing deep infection after TSA. In a
recent long-term follow-up study of total shoulder infections, male gender and younger patient age were
significantly associated with a higher infection risk. Other comorbidities and indications for TSA were not
predictive. Posterior glenoid wear results in increased retroversion and erosion of the bony vault, which
can compromise component fixation. Iannotti and associates reported on 13 patients with varying degrees
of glenoid deformity. At 20 degrees of retroversion, optimal glenoid component placement after eccentric
reaming was associated with glenoid vault perforation.

© 2014 American Academy of Orthopaedic Surgeons 2014 Shoulder and Elbow Self-Assessment Examination
2014 Shoulder and Elbow Self-Assessment Examination Answer Book • 47

RECOMMENDED READINGS FOR QUESTIONS 36 THROUGH 39


Matsen FA III, Rockwood CA Jr, Wirth MA, Lippitt SB, Parsons M. Glenohumeral arthritis and its
management. In: Rockwood CA Jr, Matsen FA III, Wirth MA, Lippitt SB, eds. The Shoulder. Vol 2. 3rd ed.
Philadelphia, PA: Saunders; 2004:879-1000.

Lee BK, Vaishnav S, Rick Hatch GF 3rd, Itamura JM. Biologic resurfacing of the glenoid with meniscal
allograft: long-term results with minimum 2-year follow-up. J Shoulder Elbow Surg. 2012 Aug 25. [Epub
ahead of print] PubMed PMID: 22929583.

Bryant D, Litchfield R, Sandow M, Gartsman GM, Guyatt G, Kirkley A. A comparison of pain, strength,
range of motion, and functional outcomes after hemiarthroplasty and total shoulder arthroplasty in patients
with osteoarthritis of the shoulder. A systematic review and meta-analysis. J Bone Joint Surg Am. 2005
Sep;87(9):1947-56. Review. PubMed PMID: 16140808.

Singh JA, Sperling JW, Schleck C, Harmsen WS, Cofield RH. Periprosthetic infections after total shoulder
arthroplasty: a 33-year perspective. J Shoulder Elbow Surg. 2012 Nov;21(11):1534-41. doi: 10.1016/j.
jse.2012.01.006. Epub 2012 Apr 18. PubMed PMID: 22516570.

Iannotti JP, Greeson C, Downing D, Sabesan V, Bryan JA. Effect of glenoid deformity on glenoid
component placement in primary shoulder arthroplasty. J Shoulder Elbow Surg. 2012 Jan;21(1):48-55.
doi: 10.1016/j.jse.2011.02.011. Epub 2011 May 20. PubMed PMID: 21600787.

END OF SERIES

Question 40
A 75-year-old woman with rheumatoid arthritis and a long history of oral corticosteroid use sustains a
comminuted intra-articular distal humerus fracture. What is the best surgical option?

1. Open reduction internal fixation (ORIF) with parallel plates


2. ORIF with orthogonal plates and iliac crest bone grafting
3. Total elbow arthroplasty (TEA)
4. Closed reduction and percutaneous pinning

PREFERRED RESPONSE: 3

DISCUSSION
TEA is the best surgical option. McKee and associates published a multicenter randomized controlled
trial comparing ORIF to TEA in elderly patients. TEA resulted in better 2-year clinical functional scores
and more predictable outcomes compared to ORIE TEA was also likely to result in a lower resurgical
rate; one-quarter of patients with fractures randomized to ORIF could not achieve stable :fixation. Further,
Frankle and associates reported a comparative study of TEA vs ORIF in 24 elderly women. TEA outcomes
were again superior to ORIF at a minimum of2 years offollowup. TEA was especially useful in patients
with comorbidities that compromise bone stock, including osteoporosis and oral corticosteroid use. Closed
reduction and percutaneous pinning studies have not been published on the adult population.

© 2014 American Academy of Orthopaedic Surgeons 2014 Shoulder and Elbow Self-Assessment Examination
48 • American Academy of Orthopaedic Surgeons

RECOMMENDED READINGS
McKee MD, Veillette CJ, Hall JA, Schemitsch EH, Wild LM, McCormack R. Perey B, Goetz T, Zomar
M, Moon K, Mandel S, Petit S, Guy P, Leung I. A multicenter, prospective, randomized, controlled trial
of open reduction--internal :fixation versus total elbow arthroplasty for displaced intra-articular distal
humeral fractures in elderly patients. J Shoulder Elbow Surg. 2009 Jan-Feb;18(1):3-12. doi: 10.1016/j.
jse.2008.06.005. Epub 2008 Sep 26. PubMed PlVllD: 18823799.

Frankle MA, Herscovici D Jr, DiPasquale TG, Vasey MB, Sanders RW. A comparison of open reduction
and internal fixation and primary total elbow arthroplasty in the treatment of intraarticular distal humerus
fractures in women older than age 65. J Orthop Trauma. 2003 Aug;l7(7):473-80. PubMed PlVllD:
12902784.

Question 41
A 67-year-old man with right shoulder osteoarthritis remains symptomatic despite a course of nonsurgical
treatment. A CT scan of the shoulder before surgery shows eccentric posterior glenoid wear with 10
degrees of retroversion. What is the appropriate treatment of this glenoid bone loss?

1. Implantation of the glenoid component in 10 degrees ofretroversion


2. Hemiarthroplasty
3. Eccentric reaming of glenoid
4. Posterior glenoid bone graft

PREFERRED RESPONSE: 3

DISCUSSION
Total shoulder arthroplasty (TSA) is superior to hemiarthroplasty for primary osteoarthritis. The most
common complication of TSA is glenoid loosening and malposition, which are common causes of glenoid
failure. Glenoid malposition decreases the glenohumeral contact area and subsequently increases contact
pressures. Altering the stem version to accommodate glenoid retroversion does not appropriately address
soft-tissue balancing. A retroversion of less than 12 degrees to 15 degrees can be corrected with eccentric
reaming without excessively compromising glenoid bone stock and risking glenoid vault penetration
by the glenoid component. Posterior glenoid bone grafting may be considered for glenoid retroversion
exceeding 15 degrees.

© 2014 American Academy of Orthopaedic Surgeons 2014 Shoulder and Elbow Self-Assessment Examination
2014 Shoulder and Elbow Self-Assessment Examination Answer Book• 49

RECOMMENDED READINGS
Levine WN, Fischer CR, Nguyen D, Flatow EL, Ahmad CS, Bigliani LU. Long-Term Follow-up
of Shoulder Hemiarthroplasty for Glenohumeral Osteoarthritis. J Bone Joint SurgAm. 2012 Nov
21;94(22):e1641-7. doi: 10.2106/JBJS.K.00603. PubMed PMID: 23172331.

Edwards TB, Kadakia NR, Boulahia A, Kempf JF, Boileau P, Nemoz C, Walch G. A comparison of
hemiarthroplasty and total shoulder arthroplasty in the treatment of primary glenohumeral osteoarthritis:
results of a multicenter study. J Shoulder Elbow Surg. 2003 May-Jun;12(3):207-13. PubMed PMID:
12851570.

Sears BW, Johnston PS, Ramsey ML, Williams GR. Glenoid bone loss in primary total shoulder
arthroplasty: evaluation and management. J Am.Acad Orthop Surg. 2012 Sep;20(9):604-13. doi: 10.5435/
JAAOS-20-09-604. Review. PubMed PMID: 22941803.

Shapiro TA, McGarry MH, Gupta R, Lee YS, Lee TQ. Biomechanical effects of glenoid retroversion in
total shoulder arthroplasty. J Shoulder Elbow Surg. 2007 May-Jun;16(3 Suppl):S90-5. Epub 2006 Dec 12.
PubMed PMID: 17169588.

Nowak DD, Bahu MJ, Gardner TR, Dyrszka MD, Levine WN, Bigliani LU, Ahmad CS. Simulation of
surgical glenoid resurfacing using three-dimensional computed tomography of the arthritic glenohumeral
joint: the amount of glenoid retroversion that can be corrected. J Shoulder Elbow Surg. 2009 Sep-
Oct;18(5):680-8. doi: 10.1016/j.jse.2009.03.019. Epub 2009 May 31. PubMed PMID: 19487133.

«:> 2014 American Academy of Orthopaedic Surgeons 2014 Shoulder and Elbow Self-Assessment Examination
50 • American Academy of Orthopaedic Surgeons

Figure 42

Question 42
Figure 42 is the MRI scan of a 52-year-old active man who fell from a ladder 6 weeks ago and sustained
an isolated glenohumeral dislocation that was reduced in the emergency department. He wore his sling
for about 2 weeks and arrived at your clinic today after referral by his primary care doctor. Examination
reveals sensation intact throughout his hand, forearm, and shoulder girdle. Belly press examination
findings are normal, but painful. He has tenderness to palpation on the anterior shoulder and a painful
speed test. Rotator cuff repair associated with tenotomy of the indicated structure will result in what
condition when compared to tenodesis of the same structure?

1. Decreased strength in forward elevation


2. Increased external rotation
3. Cosmetic deformity
4. Earlier fatigability with pronation

PREFERRED RESPONSE: 3

DISCUSSION
Patients with subluxation of the biceps tendon and full-thickness tears of the subscapularis require
treatment of the biceps tendon. Studies have shown there is increased likelihood for cosmetic "Popeye"
deformity after tenotomy when compared to tenodesis, but there is little or no functional deficit associated
with tenotomy. In other studies, there have been findings of supination strength reduction in patients with
tenotomy compared to those with tenodesis.

RECOMMENDED READINGS
Hsu AR, Ghodadra NS, Provencher MT, Lewis PB, Bach BR. Biceps tenotomy versus tenodesis: a review
of clinical outcomes and biomechanical results. J Shoulder Elbow Surg. 2011 Mar;20(2):326-32. Epub
2010 Nov 4. Review. PMID: 21051241.

Slenker NR, Lawson K, Ciccotti MG, Dodson CC, Cohen SB. Biceps tenotomy versus tenodesis: clinical
outcomes. Arthroscopy. 2012 Apr;28(4):576-82. Epub 2012 Jan 28. Review. PMID: 22284407.

© 2014 American Academy of Orthopaedic Surgeons 2014 Shoulder and Elbow Self-Assessment Examination
2014 Shoulder and Elbow Self-Assessment Examination Answer Book• 51

Question 43
A complication associated with using the Morrey approach (triceps reflecting) to implant a
semiconstrained total elbow arthroplasty is

1. loss of elbow extensor power.


2. implant dislocation.
3. implant malposition.
4. development ofheterotopic ossification.

PREFERRED RESPONSE: 1

DISCUSSION
Numerous approaches can be used to implant a total elbow arthroplasty. The Morrey approach identifies,
transposes, and protects the ulnar nerve, and then subperiosteally reflects the triceps off the ulna. The
sleeve of tissue is very thin distally, and the triceps need to be meticulously repaired at the time of closure.
Implant dislocation and malposition are less likely with an extensile approach, and dislocation is unlikely
with a semiconstrained implant. The development of heterotopic ossification is unrelated to the surgical
approach used for elbow arthroplasty.

RECOMMENDED READINGS
Kim JM, Mudgal CS, Konopka JF, Jupiter JB. Complications of total elbow arthroplasty. J Am Acad
Orthop Surg. 2011 Jun;19(6):328-39. Review. PubMed PMID: 21628644.

Lee DH. Linked total elbow arthroplasty. Hand Clin. 2011 May;27(2):199-213, vi. PubMed PMID:
21501791.

Voloshin I, Schippert DW, Kakar S, Kaye EK, Morrey BF. Complications of total elbow replacement: a
systematic review. J Shoulder Elbow Surg. 2011 Jan;20(1):158-68. Review. PubMed PMID: 21134667.

CLINICAL SITUATION FOR QUESTIONS 44 AND 45


A 19-year-old hockey player returns home from college over holiday break and experiences multiple
recurrent dislocations only 1 year after an arthroscopic stabilization.

Question 44
What is the preferred test to evaluate this patient?

1. Electromyography
2. MRI scan with intravenous contrast
3. Bone scan
4. CT arthrogram

PREFERRED RESPONSE: 4
(0 2014 American Academy of Orthopaedic Surgeons 2014 Shoulder and Elbow Self-Assessment Examination
52 • American Academy of Orthopaedic Surgeons

Question 45
The treating physician opted to perform a Latarjet coracoid transfer. What is the primary mechanism of
stability?

1. Capsular reinforcement by the coracoacromial ligament


2. Dynamic sling created by the conjoint tendon
3. Increased glenoid depth
4. Subscapularis tenodesis

PREFERRED RESPONSE: 3

DISCUSSION FOR QUESTIONS 44 AND 45


Because bone loss is likely the biggest risk factor for this patient's recurrence, a CT arthrogram will
provide the most accurate representation of the defect. An arthrogram enhances the ability to evaluate
the capsule and ligamentous attachments that may have a role in recurrent instability. Collision athletes
may have a lower recurrence rate with open surgery. The Latarjet coracoid transfer achieves its primary
stability through the increased glenoid surface area that the bone block provides.

RECOWvffiNDED READINGS FOR QUESTIONS 44 AND 45


Schmid SL, Farshad M, Catanzaro S, Gerber C. The Latarjet procedure for the treatment ofrecurrence
of anterior instability of the shoulder after operative repair: a retrospective case series of forty-nine
consecutive patients. J Bone Joint SurgAm. 2012 Jun 6;94(11):e75. doi: 10.2106/JBJS.K.00380. PubMed
PMID: 22637215.

Allain J, Goutallier D, Glorion C. Long-term results of the Latarjet procedure for the treatment of anterior
instability of the shoulder. J Bone Joint SurgAm. 1998 Jun;80(6):841-52. PubMed PMID: 9655102.

Ghodadra N, Gupta A, Romeo AA, Bach BR Jr, Verma N, Shewman E, Goldstein J, Provencher MT.
Normalization of glenohumeral articular contact pressures after Latarjet or iliac crest bone-grafting. J
Bone Joint SurgAm. 2010 Jun;92(6):1478-89. doi: 10.2106/JBJS.I.00220. PubMed PMID: 20516324.

Yamamoto N, Muraki T, Sperling JW, Steinmann SP, Cofield RH, ltoi E, An KN. Stabilizing mechanism
in bone-grafting of a large glenoid defect. J Bone Joint Surg Am. 2010 Sep 1;92(11):2059-66. doi:
10.2106/JBJS.I.00261. PubMed PMID: 20810855.

Thomazeau H, Courage 0, Barth J, Pelegri C, Charousset C, Lespagnol F, Nourissat G, Audebert S, Guillo


S, Toussaint B, Lafosse L, Brade! J, Veillard D, Boileau P; French Arthroscopy Society. Can we improve
the indication for Bankart arthroscopic repair? A preliminary clinical study using the ISIS score. Orthop
Traumatol Surg Res. 2010 Dec;96(8 Suppl):S77-83. doi: 10.1016/j.otsr.2010.09.007. Epub 2010 Oct 28.
PubMed PMID: 21035419.

END OF SERIES

© 2014 American Academy of Orthopaedic Surgeons 2014 Shoulder and Elbow Self-Assessment Examination
2014 Shoulder and Elbow Self-Assessment Examination Answer Book • 53

Question 46
A 45-year-old woman has a 3-month history of left shoulder pain. She has tried 2 months of physical
therapy focused on rotator cuff strengthening without experiencing relief. A subacromial corticosteroid
injection fails to provide lasting relief. Examination reveals no atrophy or winging. She has anterior and
posterior shoulder tenderness, full symmetric forward elevation and abduction, and pain with maximal
passive forward elevation. She has pain with internal rotation in 90 degrees of forward elevation. She
has an increased distance between the antecubital fossa and coracoid process with cross chest adduction
compared to the contralateral side. No weakness is appreciated. Radiographs reveal a type II acromion.
What is the best next step?

1. Posterior capsular stretching


2. Arthroscopic subacromial decompression
3. Diagnostic acromioclavicular (AC) joint injection
4. MRI scan

PREFERRED RESPONSE: 1

DISCUSSION
This patient's examination is consistent with posterior capsular tightness, which can mimic impingement.
Four weeks of posterior capsular stretches will likely improve motion and pain. Surgical treatment should
be considered only after failure of a dedicated stretching program. An AC joint injection would help
differentiate this condition from AC joint arthritis, but in light of the radiographic findings, arthritis is
unlikely. An MRI scan likely will not change the initial treatment at this point.

RECO1\1MENDED READINGS
Bach HG, Goldberg BA. Posterior capsular contracture of the shoulder. J Am Acad Orthop Surg. 2006
May;l4(5):265-77. Review. PubMed PMID: 16675620.

Papadonikolakis A, McKenna M, Warme W, Martin BI, Matsen FA 3rd. Published evidence relevant to the
diagnosis of impingement syndrome of the shoulder. J Bone Joint SurgAm. 2011 Oct 5;93(19):1827-32.
doi: 10.2106/JBJS.J.0l748. Review. PubMed PMID: 22005869.

CLINICAL SITUATION FOR QUESTIONS 47 THROUGH 49


A 13-year-old pitcher reports the immediate onset of medial elbow pain after throwing a pitch. Upon
examination, the patient is tender to palpation at the medial epicondyle and has pain and instability with
valgus testing of the elbow.

<D 2014 American Academy of Orthopaedic Surgeons 20I4 Shoulder and Elbow Self-Assessment Examination
54 • American Academy of Orthopaedic Surgeons

Question 47
Which is the most appropriate diagnostic test?

1. MRI arthrogram
2. CT scan with 3-dimensional reconstructions
3. Plain radiographs of both elbows
4. Ultrasound

PREFERRED RESPONSE: 3

Question 48
Why was your response for question 32-1 the most appropriate test for this patient?

1. To evaluate for apophyseal injury


2. To evaluate for osteochondral defect
3. To evaluate for hematoma
4. To evaluate for valgus overload injury

PREFERRED RESPONSE: 1

Question 49
If the patient were a college pitcher with a similar presentation and examination, what structure would
most likely be injured?

1. Ulnar collateral ligament


2. Pronator teres
3. Ligament of Struthers
4. Lateral collateral ligament

PREFERRED RESPONSE: 1

DISCUSSION FOR QUESTIONS 47 THROUGH 49


The patient has an acute avulsion fracture of the medial epicondyle, which can occur in response
to the valgus load placed on the elbow while throwing. Diagnosis is confirmed by radiograph, with
comparison views of the uninjured elbow to evaluate for physeal closure vs injury. In older pitchers, the
ulnar collateral ligament fails rather than the bone of the medial epicondyle. Advanced imaging may be
necessary to confirm the diagnosis of an ulnar collateral ligament injury and/or bony injury.

© 20I4 American Academy of Orthopaedic Surgeons 2014 Shoulder and Elbow Self-Assessment Examination
2014 Shoulder and Elbow Self-Assessment Examination Answer Book• 55

RECOMMENDED READINGS FOR QUESTIONS 47 THROUGH 49


Osbahr DC, Chalmers PN, Frank JS, Williams RJ 3rd, Widmann RF, Green DW. Acute, avulsion fractures
of the medial epicondyle while throwing in youth baseball players: a variant of Little League elbow. J
Shoulder Elbow Surg. 2010 Oct;19(7):951-7. doi: 10.1016/j.jse.2010.04.038. Epub 2010 Aug 5. PubMed
P:MID: 20688542.

Gottschalk HP, Eisner E, Hosalkar HS. Medial epicondyle fractures in the pediatric population. J Am
Acad Orthop Surg. 2012 Apr;20(4):223-32. doi: 10.5435/JAAOS-20-04-223. Review. PubMed P:MID:
22474092.

END OF SERIES

Question 50
A 65-year-old woman has electrodiagnostic findings of ulnar nerve entrapment at the elbow. You counsel
the patient that

1. medial epicondylectomy and submuscular transposition is the preferred treatment.


2. arthroscopic decompression is associated with lower complication rates compared to open
treatment.
3. simple decompression can be as effective as ulnar nerve transposition.
4. duration of symptoms is the most important predictor of outcome.

PREFERRED RESPONSE: 3

DISCUSSION
Multiple studies have demonstrated that simple ulnar nerve decompression is as effective as
subcutaneous transposition for most symptomatic ulnar neuropathy. Ulnar nerve instability before or
after decompression may best be treated by transposition rather than simple decompression. Submuscular
transposition with or without medial epicondylectomy may be best reserved for revision surgery or
patients who are exceedingly thin. Arthroscopic nerve decompression has been reported with arthroscopic
treatment of elbow arthritis, but is associated with a higher complication and revision rate than the
standard techniques. Adequate nerve decompression, rather than duration of symptoms, is the most
important predictor of outcome.

© 2014 American Academy of Orthopaedic Surgeons 2014 Shoulder and Elbow Self-Assessment Examination
56 • American Academy of Orthopaedic Surgeons

RECOMMENDED READINGS
Caliandro P, La Torre G, Padua R, Giannini F, Padua L. Treatment for ulnar neuropathy at the elbow.
Cochrane Database Syst Rev. 2012 Jul 11;7:CD006839. doi: 10.1002/14651858.CD006839.pub3. Review.
PubMed PMID: 22786500.

Zlowodzki M, Chan S, Bhandari M, Kalliainen L, Schubert W. Anterior transposition compared with


simple decompression for treatment of cubital tunnel syndrome. A meta-analysis of randomized,
controlled trials. J Bone Joint SurgAm. 2007 Dec;89(12):2591-8. PubMed PMID: 18056489.

Macadam SA, Gandhi R, Bezuhly M, Lefaivre KA. Simple decompression versus anterior subcutaneous
and submuscular transposition of the ulnar nerve for cubital tunnel syndrome: a meta-analysis. J Hand
SurgAm. 2008 Oct;33(8):1314.el-12. doi: 10.1016/j.jhsa.2008.03.006. Review. PubMed PMID:
18929194.

Gellman H. Compression of the ulnar nerve at the elbow: cubital tunnel syndrome. Instr Course Leet.
2008;57:187-97. Review. PubMedPMID: 18399580.

Kovachevich R, Steinmann SP. Arthroscopic ulnar nerve decompression in the setting of elbow
osteoarthritis. J Hand SurgAm. 2012 Apr;37(4):663-8. doi: 10.1016/j.jhsa.2012.01.003. Epub 2012 Mar 3.
PubMed PMID: 22386545.

Shi Q, MacDermid JC, Santaguida PL, Kyu HH. Predictors of surgical outcomes following anterior
transposition of ulnar nerve for cubital tunnel syndrome: a systematic review. J Hand Surg Am. 2011
Dec;36(12):1996-2001.el-6. doi: 10.1016/j.jhsa.2011.09.024. Review. PubMed PMID: 22123047.

Question 51
A 21-year-old college swimmer presents with an inability to compete for longer than 1 year because of
right shoulder pain and subjective symptoms of instability despite physical therapy. Recent radiographs
and an MRI scan of her shoulder demonstrate an intact labral complex. Her symptoms are reproduced with
sulcus testing and load and shift maneuvers in both anterior and posterior directions. What is the most
appropriate next treatment step?

1. Continued physical therapy


2. Open capsular shift
3. Arthroscopic capsulolabral shift
4. Thermal capsulorrhaphy

PREFERRED RESPONSE: 2

© 2014 American Academy of Orthopaedic Surgeons 2014 Shoulder and Elbow Self-Assessment Examination
2014 Shoulder and Elbow Self-Assessment Examination Answer Book• 57

DISCUSSION
In this scenario, the patient has already failed therapy. An MRI scan did not indicate recurrent injury to
the labrum. The open capsular shift procedure initially popularized by Neer and modified by Wirth and
Rockwood allows surgeons to reduce joint volume by imbricating the patulous inferior capsule through an
anterior axillary-based incision. Through this anterior approach, concomitant injuries such as a humeral
avulsion of the glenohumeral ligament, Bankart lesions, and subscapularis tears may be addressed.
Multidirectional instability (MDI) of the shoulder is defined as symptomatic instability in more than 1
direction. Both dynamic (rotator cuff, periscapular muscles) and static stabilizers (glenoid, labrum, and
joint capsule) impart stability to the glenohumeraljoint. In this patient, the examination may indicate a
positive sulcus sign, increased humeral head translation in more than 1 direction with the load-and-shift
test, and generalized hyperlaxity such as elbow, knee, and metacarpophalangeal joint hyperextension.
Radiographs may infrequently reveal glenoid dysplasia and bone loss. MRI arthrogram imaging may
demonstrate a redundant capsule without specific injury to the labral complex. Initial management of
MDI involves therapy with a focus on rotator cuff and periscapular muscle strengthening. Arthroscopic
capsulorrhaphy may be a viable option but is not indicated in this scenario based upon lack of injury to the
labrum. As with the operi capsular shift, care must be taken when performing an arthroscopic plication to
avoid overreduction and subsequent shoulder stiffness. Thermal caspulorrhaphy has been abandoned as a
treatment option because of high failure rate (60%) and risk for chondrolysis and axillary nerve injury.

RECO:rvIMENDED READINGS
Schenk TJ, Brems JJ. Multidirectional instability of the shoulder: pathophysiology, diagnosis, and
management. J Am Acad Orthop Surg. 1998 Jan-Feb;6(1):65-72. Review. PubMed PMID: 9692942.

Gaskill TR, Taylor DC, Millett PJ. Management ofmultidirectional instability of the shoulder. J Am Acad
Orthop Surg. 2011 Dec;l9(12):758-67. Review. PubMed PMID: 22134208.

Bois AJ, Wirth MA. Revision open capsular shift for atraumatic and multidirectional instability of the
shoulder. J Bone Joint SurgAm. 2012Apr 18;94(8):748-56. PubMed PMID: 22517392.

Question 52
A 15-year-old girl has experienced 6 months of increasing dominant shoulder pain while playing
volleyball. Her pain is so significant that she can no longer compete. Examination demonstrates 190
degrees of forward elevation, 110 degrees of external rotation at the side, and internal rotation up the
back to T2 bilaterally. She also has 15 degrees of bilateral elbow hyperextension. Load and shift testing
demonstrates pain with anterior and posterior drawer tests. She has a large sulcus on examination that
causes pain during testing. Forward elevation and external rotation strength testing shows 4/5 strength.
There is no scapular winging and radiograph findings are normal. What is the best next step?

1. Physical therapy for rotator cuff strengthening


2. Subacromial corticosteroid injection
3. MRI arthrogram
4. Arthroscopic stabilization

PREFERRED RESPONSE: 1

© 2014 American Academy of Orthopaedic Surgeons 2014 Shoulder and Elbow Self-Assessment Examjnation
58 • American Academy of Orthopaedic Surgeons

DISCUSSION
This patient has multidirectional instability as evidenced by her hyperlaxity and excessive range of
motion. Patients with pain after activities often have weak rotator cuff musculature and improve with
strengthening of the rotator cuff and proprioceptive retraining. Subacromial injection likely cannot help
this patient because it will not treat her underlying motor weakness in the rotator cuff or her dyskinesia.
MRI arthrogram is not indicated unless she fails nonsurgical treatment. Arthroscopic stabilization also
would be reserved for patients who fail nonsurgical treatment.

RECOMMENDED READINGS
Gaskill TR, Taylor DC, Millett PJ. Management of multidirectional instability of the shoulder. J Am Acad
Orthop Surg. 2011 Dec;19(12):758-67. Review. PMID: 22134208.

Jacobson ME, Riggenbach M, Wooldridge AN, Bishop JY. Open capsular shift and arthroscopic capsular
plication for treatment ofmultidirectional instability. Arthroscopy. 2012 Jul;28(7):1010-7. Review. PMID:
22365265.

Figure 53a Figure 53b

CLINICAL SITUATION FOR QUESTIONS 53 AND 54


Figures 53a and 53b are the radiographs of a 47-year-old right-hand-dominant active man with a 10-year
history of progressive right elbow pain associated with stiffness. He previously underwent collateral
ligament reconstruction. He has pain throughout his range of motion arc, which currently measures
20 degrees of extension to 80 degrees offiexion. Initial treatment with nonsteroidal anti-inflammatory
medication, physical therapy, cortisone injections, and arthroscopic debridement has failed to provide
relief of his symptoms and improvement in function.

© 2014 American Academy of Orthopaedic Surgeons 2014 Shoulder and Elbow Self-Assessment Examination
2014 Shoulder and Elbow Self-Assessment Examination Answer Book• 59

Question 53
What is the most appropriate next treatment step for this patient?

1. Total elbow arthroplasty (TEA)


2. Distal humeral replacement arthroplasty
3. Arthroscopic release with debridement
4. Soft-tissue interposition arthroplasty

PREFERRED RESPONSE: 4

Question 54
What is the most appropriate treatment if instability is present at the time of evaluation?

1. TEA
2. Distal humeral replacement arthroplasty
3. Arthroscopic release with debridement
4. Soft-tissue interposition arthroplasty

PREFERRED RESPONSE: 1

DISCUSSION FOR QUESTIONS 53 AND 54


The radiographs reveal ulnohumeral arthrosis with relative sparing of the radiocapitellar articulation
secondary to underlying osteoarthritis.Arthrosis of the elbow joint in this young and active patient
presents a treatment dilemma for the surgeon. Interposition arthroplasty allows for improved function
with pain relief and no weight-lifting restrictions, as required with TEA. This option is an intermediate
procedure that preserves bone stock and allows for conversion to a TEA if necessary. Conventional TEA
would provide pain relief with improved range of motion, but activity limitation and lifetime weight
restrictions make this an undesirable option. Arthroscopic debridement is not an option, considering the
previous failure from this modality. Contraindications for soft-tissue interposition arthroplasty include
elbow instability, active infection, and pain without motion loss. Common complications associated with
this procedure include instability, infection, ulnar neuropathy, bone resorption, and heterotopic bone
formation.

© 2014 American Academy of Orthopaedic Surgeons 2014 Shoulder and Elbow Self-Assessment Examination
60 • American Academy of Orthopaedic Surgeons

RECOMMENDED READINGS FOR QUESTIONS 53 AND 54


Larson AN, Morrey BF. Interposition arthroplasty with an Achilles tendon allograft as a salvage procedure
for the elbow. J Bone Joint SurgAm. 2008 Dec;90(12):2714-23. PubMed PMID: 19047718.

Blaine TA, Adams R, Morrey BF. Total elbow arthroplasty after interposition arthroplasty for elbow
arthritis. J Bone Joint SurgAm. 2005 Feb;87(2):286-92. PubMed PMID: 15687149.

Celli A, Morrey BF. Total elbow arthroplasty in patients forty years of age or less. J Bone Joint Surg Am.
2009 Jun;91(6):1414-8. PubMed PMID: 19487519.

Surgical management of traumatic conditions of the elbow. In: Weisel S, Ramsey ML, eds. Operative
Techniques in Orthopaedic Surgery. Philadelphia, PA: Lippincott, Williams & Wilkins; 2011:3453-3461.

END OF SERIES

Figure 55a Figure 55b

Question 55
Figures 55a and 55b are the radiographs of a 64-year-old woman with a history significant for rheumatoid
arthritis who has the chief complaint of right elbow pain. She has been treated with tumor necrosis factor-
alpha inhibitors and oral corticosteroids for several years. The patient experiences severe global elbow
pain and crepitus. What process primarily is responsible for joint destruction in rheumatoid arthritis?

1. Traumatic insult resulting in complement activation


2. Mutation in the rheumatoid factor gene
3. Osteoblast paracrine signaling resulting in proteolytic collagen degradation
4. Inflammation resulting in a hyperplastic synovial joint lining

PREFERRED RESPONSE: 4

© 2014 American Academy of Orthopaedic Surgeons 2014 Shoulder and Elbow Self-Assessment Examination
2014 Shoulder and Elbow Self-Assessment Examination Answer Book • 61

DISCUSSION
Rheumatoid arthritis is a systemic inflammatory disorder marked by erosive arthritis in multiple joints.
Elbow involvement is common. The pathologic lesion in rheumatoid arthritis is pannus, a hyperplastic
synovial proliferation that ultimately results in proteoglycan and collagen digestion. Rheumatoid factor
mutations, traumatic insults resulting in complement activation and osteoblast paracrine signaling, are
not involved in the pathologic process. The Larsen classification assesses the progression of rheumatoid
changes in the elbow. Stage I is characterized by osteopenia without joint space narrowing. Stage II
indicates joint space narrowing, but a normal joint contour. Stage III is marked by joint space loss. This
patient has stage IV disease, as seen by the advanced erosive changes with trochlear groove deepening and
resulting deformity. Stage V is ankylosis.

RECOMMENDED READINGS
Chen AL, Joseph TN, Zuckerman JD. Rheumatoid arthritis of the shoulder. J Am Acad Orthop Surg. 2003
Jan-Feb;ll(l):12-24. Review. PubMed PMID: 12699368.

Larsen A, Dale K, Eek M. Radiographic evaluation of rheumatoid arthritis and related conditions by
standard reference films. Acta Radiol Diagn (Stockh). 1977 Jul;18(4):481-91. PubMed PMID: 920239.

Gill DR, Morrey BF. The Coonrad-Morrey total elbow arthroplasty in patients who have rheumatoid
arthritis. A ten to fifteen-year follow-up study. J Bone Joint Surg Am. 1998 Sep;80(9):1327-35. PubMed
PMID: 9759818.

Figure 56

CLINICAL SITUATION FOR QUESTIONS 56 THROUGH 58


Figure 56 is the radiograph of a 47-year-old woman who has pain and difficulty raising her arm
after playing 36 holes of golf in a weekend. She denies prior episodes of shoulder pain, and now has
difficulty sleeping. Examination demonstrates guarding with any shoulder motion, tenderness around the
superolateral shoulder, and normal sensory findings.

© 20 I 4 American Academy of Orthopaedic Surgeons 2014 Shoulder and Elbow Self-Assessment Examination
62 • American Academy of Orthopaedic Surgeons

Question 56
An MRI arthrogram scan of her shoulder would show

1. increased T2 signal in the rotator cuff.


2. fluid escape into the subacromial space.
3. fluid in the glenoid/labral fissure.
4. isointense signal to the rotator cuff.

PREFERRED RESPONSE: 4

Question 57
The lesion indicated in the image is made of

1. calcium carbonate apatite.


2. hyperproliferative white blood cells.
3. hydroxyapatite crystals.
4. degenerated tenocytes.

PREFERRED RESPONSE: 1

Question 58
The best initial treatment would entail

1. physical therapy and nonsteroidal anti-inflammatory medications.


2. open biopsy of the lesion for permanent section.
3. manipulation under anesthesia.
4. shoulder arthroscopy.

PREFERRED RESPONSE: 1

DISCUSSION FOR QUESTIONS 56 THROUGH 58


Calcific tendinitis of the shoulder is a deposition of calcium carbonate apatite crystals into the structure of
the rotator cuff tendon. The crystalline form appears to progress throughout the clinical disease process,
demonstrating increasing matured stoichiometric apatite deposition during the resorptive phase. MRI can
be difficult to interpret because the signal of the calcific lesion is frequently similar to that seen in normal
supraspinatus tendon. Plain radiographs remain the gold standard for diagnosis. Ultrasound can be a useful
ancillary study to determine the location and size of the lesion. Primary management of calcific tendinitis
starts with nonsurgical treatment including physiotherapy and injections, if indicated. Mixed results have
been reported with extracorporeal shock wave therapy. Surgical removal with repair of the tendon in larger
lesions remains the definitive treatment when nonsurgical modalities fail. Subacromial decompression
may improve pain relief in patients who require surgery; however, patients with decompression may take
longer to fully recover.

© 2014 American Academy of Orthopaedic Surgeons 2014 Shoulder and Elbow Self-Assessment Examination
2014 Shoulder and Elbow Self-Assessment Examination Answer Book• 63

RECOMMENDED READINGS FOR QUESTIONS 56 THROUGH 58


Zubler C, Mengiardi B, Schmid MR, Hodler J, Jost B, Pfirrmann CW. MR arthrography in calcific
tendinitis of the shoulder: diagnostic performance and pitfalls. Eur Radiol. 2007 Jun;l7(6):1603-10. Epub
2006 Oct 12. P:tvfID: 17036154.

Oliva F, Via AG, Maffulli N. Physiopathology of intratendinous calcific deposition. BMC Med. 2012 Aug
23;10:95. doi: 10.1186/1741-7015-10-95. PubMed PMID: 22917025.

Ark JW, Flock TJ, Flatow EL, Bigliani LU. Arthroscopic treatment of calcific tendinitis of the shoulder.
Arthroscopy. 1992;8(2):183-8. PubMed PMID: 1637430.

Marder RA, Heiden EA, Kim S. Calcific tendonitis of the shoulder: is subacromial decompression in
combination with removal of the calcific deposit beneficial? J Shoulder Elbow Surg. 2011 Sep;20(6):955-
60. Epub 2011 Feb 1. PMID: 21277805.

Balke M, Bielefeld R, Schmidt C, Dedy N, Liem D. Calcifying tendinitis of the shoulder: midterm
results after arthroscopic treatment. Am J Sports Med. 2012 Mar;40(3):657-61. Epub 2011 Dec 8. PMID:
22156173.

END OF SERIES

© 2014 American Academy of Orthopaedic Surgeons 2014 Shoulder and Elbow Self-Assessment Examination
64 • American Academy of Orthopaedic Surgeons

Figure 59

Question 59
Figure 59 is the MRI scan of a 30-year-old fire fighter who dislocated his left shoulder during work
activities. His shoulder was reduced in the emergency department. After 8 weeks of physical therapy, he
continues to have apprehension when lifting and pushing the fire hose back into the truck. He has normal
rotator cuff strength and a negative sulcus sign. What treatment option will allow this patient to return to
work as soon as possible?

1. Strengthening in physical therapy


2. Anterior labral repair
3. Posterior labral repair
4. Coracoid transfer

PREFERRED RESPONSE: 3

DISCUSSION
The MRI scan shows a posterior labral tear, which is contributing to his posterior instability. If a patient
does not improve after 8 weeks of physical therapy, the therapy likely will not correct his or her instability.
Activities that involve internal rotation of the shoulder and adduction will put the shoulder at most risk for
posterior dislocation or subluxation. The patient elevates his risk when he pushes fire hoses into the truck
repetitively, which further supports the diagnosis of posterior instability attributable to a posterior labral
tear. An anterior labral repair or a coracoid transfer would not treat posterior instability.

RECOrvtMENDED READINGS
Lenart BA, Sherman SL, Mall NA, Gochanour E, Twigg SL, Nicholson GP. Arthroscopic repair for
posterior shoulder instability. Arthroscopy. 2012 Oct;28(10):1337-43. doi: 10.1016/j.arthro.2012.03.011.
Epub 2012 Jun 15. PubMed PMID: 22705299.

Provencher MT, LeClere LE, King S, McDonald LS, Frank RM, Mologne TS, Ghodadra NS, Romeo AA.
Posterior instability of the shoulder: diagnosis and management. Am J Sports Med. 2011 Apr;39(4):874-
86. doi: 10.ll 77/0363546510384232. Epub 2010 Dec 4. PubMed PMID: 21131678.

© 2014 American Academy of Orthopaedic Surgeons 2014 Shoulder and Elbow Self-Assessment Examination
2014 Shoulder and Elbow Self-Assessment Examination Answer Book• 65

CLINICAL SITUATION FOR QUESTIONS 60 AND 61


A 10-year-old left-hand-dominant baseball pitcher has had left elbow pain for 6 weeks. His pain primarily
is located medially, and he states that it is worst during the late cocking/early acceleration phase of his
pitch. Recently he noticed that he is not able to throw as fast as usual. He decreased his pitch count by
half during the last 2 weeks without significant improvement in his symptoms. When he is not pitching, he
does not have significant pain. Radiographs show widening of the medial epicondyle physis.

Question 60
At what age does the medial epicondyle epiphysis ossification center appear and then fuse?

1. Appears at 2 to 3 years, fuses at 12 to 13 years


2. Appears at 5 to 6 years, fuses at 15 to 16 years
3. Appears at 6 to 8 years, fuses at 12 to 13 years
4. Appears at 8 to 10 years, fuses at 15 to 16 years

PREFERRED RESPONSE: 2

Question 61
What biomechanical forces and pathology most likely underlie this patient's pain and injury?

1. Acute avulsion of the medial epicondyle attributable to valgus stress


2. Chronic weakening of the ulnar collateral ligament attributable to chronic tension forces
3. Chronic compressive forces on the medial epicondyle leading to fragmentation
4. Chronic tension forces ofvalgus overload on the medial epicondyle leading to physeal
separation

PREFERRED RESPONSE: 4

DISCUSSION FOR QUESTIONS 60 AND 61


The medial epicondyle ossification center is first seen at 5 to 6 years of age and is the last to fuse at age
15 to 16. The capitellum first appears at age 1 to 2. The radial epicondylar epiphysis appears at 2 to 4
years. The trochlea appears at age 8 to 10; the olecranon appears at approximately age 10. The lateral
epicondylar epiphysis is the last to appear at age 12.

Repetitive tensile stress on the medial epicondyle is caused by the flexor-pronator mass and the ulnar
collateral ligament. This chronic valgus microtrauma eventually can lead to apophysitis and/or stress
fracture. In the skeletally immature athlete, tension across the medial elbow produces a physeal injury
rather than a ligamentous injury. Based upon the history, this is a repetitive overuse injury and not an acute
injury.

© 2014 American Academy of Orthopaedic Surgeons 2014 Shoulder and Elbow Self-Assessment Examination
66 • American Academy of Orthopaedic Surgeons

RECOMMENDED READINGS FOR QUESTIONS 60 AND 61


Klingele KE, Kocher MS. Little league elbow: valgus overload injury in the pediatric athlete. Sports Med.
2002;32(15):1005-15. Review. PubMed PMID: 12457420.

Rudzki JR, Paletta GA Jr. Juvenile and adolescent elbow injuries in sports. Clin Sports Med. 2004
Oct;23(4):581-608, ix. Review. PubMed PMID: 15474224.

END OF SERIES

Question 62
A 35-year-old man fell off of a roof and sustained an extra-articular supracondylar elbow fracture. He had
normal sensation in all fingers after the injury and before undergoing surgery to repair the fracture. The
ulnar nerve was not transposed, but it was inspected prior to wound closure. Ten days after surgery, the
patient has numbness in his small finger and is unable to cross his fingers. His elbow range of motion is
between 40 degrees and 100 degrees. What is the next appropriate treatment step?

I. Elbow splint at 40 degrees at night for 6 weeks


2. Electromyography (EMG)
3. Exploration of the ulnar nerve and transposition
4. Observation

PREFERRED RESPONSE: 4

DISCUSSION
This patient has an early postsurgical ulnar nerve palsy. The causes of this injury are laceration of the
nerve during surgery, entrapment of the nerve in the fracture or hardware, or traction injury during surgery.
If the orthopaedic surgeon is sure that the nerve was not lacerated at the end of the case or entrapped in the
hardware, then the nerve is probably intact and will recover. Observation is the best treatment in this case
because the nerve was checked before wound closure. Elbow splinting has not been shown to help with
postsurgical nerve recovery. EMG findings may not be accurate this early in the injury.

RECOMMENDED READINGS
Shin R, Ring D. The ulnar nerve in elbow trauma. J Bone Joint SurgAm. 2007 May;89(5):1108-16.
Review. PubMed PMID: 17473151.

Faierman E, Wang J, Jupiter JB. Secondary ulnar nerve palsy in adults after elbow trauma: a report of two
cases. J Hand SurgAm. 2001 Jul;26(4):675-8. PubMed PMID: I1466643.

© 2014 American Academy of Orthopaedic Surgeons 2014 Shoulder and Elbow Self-Assessment Examination
2014 Shoulder and Elbow Self-Assessment Examination Answer Book• 67

Question 63
A 54-year-old pipefitter falls from a ladder at work and dislocates his nondominant shoulder. His MRI
scan shows supraspinatus and infraspinatus tears with retraction to the glenoid. He cannot actively raise
his arm away from his side. He denies prior shoulder symptoms before his fall. Three weeks of physical
therapy have failed to improve his function. You and the patient decide to proceed with surgical repair.
Which is a risk factor for a poor outcome?

1. The patient's age


2. The patient's gender
3. Work-related injury
4. Acute nature of the tear

PREFERRED RESPONSE: 3

DISCUSSION
Several studies have demonstrated that patients with work-related injuries do not do as well as those
whose injuries are not work-related after repair of the rotator cuff. This patient's age and gender are not
negative prognostic indicators. The acute nature of the tear does not lead to an inferior outcome.

RECOMMENDED READINGS
Kemp KA, Sheps DM, Luciak-Corea C, Styles-Tripp F, Buckingham J, Beaupre LA. Systematic review
ofrotator cuff tears in workers' compensation patients. Occup Med (Lond). 2011 Dec;61(8):556-62. Epub
2011 Oct 19. Review. PMID: 22016341.

Bhatia S, Piasecki DP, Nho SJ, Romeo AA, Cole BJ, Nicholson GP, Boniquit N, Verma NN. Early
return to work in workers' compensation patients after arthroscopic full-thickness rotator cuff repair.
Arthroscopy. 2010 Aug;26(8):1027-34. Epub 2010 Jun 3. PMID: 20678699.

RESPONSES FOR QUESTIONS 64 THROUGH 68


1. Internal rotation stretching, core stability exercises, and scapular stabilization exercises
2. Arthroscopic debridement
3. Arthroscopic debridement with subacromial decompression
4. Arthroscopic transtendinous repair
5. Arthroscopic tear completion and repair

Please match the interventions above to the scenarios below.

© 2014 American Academy of Orthopaedic Surgeons 2014 Shoulder and Elbow Self-Assessment Examination
68 • American Academy of Orthopaedic Surgeons

Question 64
A 23-year-old Division 1 baseball pitcher is experiencing worsening pain despite completion of an
extensive, but unsuccessful, sport-specific physical therapy regimen. An MRI scan shows articular surface
tearing of the rotator cuff and internal impingement on abduction external rotation views.

PREFERRED RESPONSE: 2

Question 65
A 55-year-old woman with a bursal-sided tear less than 20% thickness and lateral acromial impingement
has failed physical therapy.

PREFERRED RESPONSE: 3

Question 66
A 55-year-old man with worsening night pain has no history of trauma, and examination demonstrates
posterior capsular tightness and scapular dyskinesia. Radiograph :findings appear normal.

PREFERRED RESPONSE: 1

Question 67
A 17-year-old high school pitcher has increasing pain accompanied by decreased ball velocity.

PREFERRED RESPONSE: 1

Question 68
A 65-year-old man who has failed nonsurgical treatment demonstrates a partial-thickness supraspinatus
tendon tear of 70% thickness.

PREFERRED RESPONSE: 5

IO 2014 American Academy of Orthopaedic Surgeons 2014 Shoulder and Elbow Self-Assessment Examination
2014 Shoulder and Elbow Self-Assessment Examination Answer Book • 69

DISCUSSION FOR QUESTIONS 64 THROUGH 68


For the young athlete, a careful examination of scapular mechanics, core stability, and internal rotation
deficits is important to diagnose, prevent, and treat a painful shoulder. Correction of the capsular
contracture and core imbalance often is enough to alleviate symptoms. If nonsurgical management fails,
MRI scan :findings and arthroscopic examination often show partial-thickness tearing of the articular
surface of the supraspinatus or infraspinatus with or without associated internal impingement or aberrant
contact with the posterior superior labrum and glenoid. The prevalence may be as high as 20% to 40%
in the overhead athletic population, likely resulting from repetitive microtrauma. Degenerative tears
often become symptomatic with an increase in size; the mechanical linkage between the supraspinatus
and infraspinatus makes it likely that the remaining fibers bear more strain as tear size increases.
Simple acromioplasty may be successful only in cases of definitive contact between the acromion
and supraspinatus. Transtendinous repairs are effective treatment for partial-thickness tears of the
supraspinatus, but they may have a slower functional recovery and a higher rate of stiffness than excision
and repair; this may be attributable, in part, to the natural overlap of the infraspinatus tendon over the
supraspinatus tendon. An "all-inside" technique may be preferable in younger patients because it is
possible to reinsert only the surface fibers that are tom, avoiding constraint of the superficial, bursa!fibers.
For tears of more than 50% thickness, completing the tear to excise the remaining degenerative fibers may
be the preferred treatment.

RECOMMENDED READINGS FOR QUESTIONS 64 THROUGH 68


Ide J, Maeda S, Takagi K. Arthroscopic transtendon repair of partial-thickness articular-side tears of the
rotator cuff: anatomical and clinical study. Am J Sports Med. 2005 Nov;33(1l ):1672-9. Epub 2005 Aug
10. PubMed PMID: 16093533.

Yang S, Park HS, Flores S, Levin SD, Makhsous M, Lin F, Koh J, Nuber G, Zhang LQ. Biomechanical
analysis ofbursal-sided partial thickness rotator cuff tears. J Shoulder Elbow Surg. 2009 May-
Jun;18(3):379-85. doi: 10.1016/j.jse.2008.12.011. Epub 2009 Mar 9. PubMed PMID: 19269860.

Finnan RP, Crosby LA. Partial-thickness rotator cuff tears. J Shoulder Elbow Surg. 2010 Jun;l9(4):609-
16. doi: 10.1016/j.jse.2009.10.017. Epub 2010 Feb 19. Review. PubMed PMID: 20171904.

Karnath G, Galatz LM, Keener JD, Teefey S, Middleton W, Yamaguchi K. Tendon integrity and functional
outcome after arthroscopic repair of high-grade partial-thickness supraspinatus tears. J Bone Joint Surg
Am. 2009 May;91(5):1055-62. doi: 10.2106/JBJS.G.00118. Erratum in: J Bone Joint SurgAm. 2009
Aug;91(8):1995. PubMed PMID: 19411453.

END OF SERIES

Cl 2014 American Academy of Orthopaedic Surgeons 2014 Shoulder and Elbow Self-Assessment Examination
70 • American Academy of Orthopaedic Surgeons

Question 69
A 17-year-old left-hand-dominant gymnast has a 10-week history of gradually progressive right
shoulder pain. She reports the onset of pain to be associated with an increase in her training regimen
while preparing for an upcoming regional competition, and denies any specific trauma to her shoulder.
Examination reveals end-range discomfort, but normal active and passive range of motion. Her
periscapular musculature strength is normal, but she demonstrates mild medial scapular winging with arm
elevation. She has 20 degrees' elbow recurvatum, a positive sulcus examination, and can hyperextend
the metacarpophalangeal joint of her index finger to 105 degrees. What is the most appropriate initial
treatment?

1. Physical therapy referral for rotator cuff and periscapular conditioning


2. Electromyography
3. Subacromial injection
4. Arthroscopic capsular plication

PREFERRED RESPONSE: 1

DISCUSSION
This patient has shoulder pain and an underlying diagnosis of multidirectional glenohumeral laxity.
There remains controversy as to whether athletes with features of generalized laxity are at increased risk
for shoulder pain. This athlete's presentation is typical in that the onset was atraumatic and associated
with an increase in her training regimen. The most appropriate treatment step is rest from competition
and institution of a dedicated physical therapeutic exercise program that emphasizes rotator cuff and
periscapular strengthening, with a focus on the serratus anterior. Electromyography would not be helpful
because the patient's isolated periscapular motor function is intact and she demonstrates a typical pattern
of acquired scapular dyskinesis seen in painful overhead athletic shoulder. Subacromial injection therapy
is not indicated in the setting of multidirectional instability. Arthroscopic surgical options are considered
as a final treatment intervention after nonsurgical measures have failed, and appropriate imaging, such as
MRI scan, has been obtained to determine the presence or absence of significant structural abnormalities.

RECOMMENDED READINGS
Bak K. The practical management of swimmer's painful shoulder: etiology, diagnosis, and treatment. Clin
J Sport Med. 2010 Sep;20(5):386-90. doi: 10.1097/JSM.0b013e3181f205fa. PubMed PMID: 20818199.

Madsen PH, Bak K, Jensen S, Welter U. Training induces scapular dyskinesis in pain-free competitive
swimmers: a reliability and observational study. Clin J Sport Med. 2011 Mar;21(2):109-13. doi: 10.1097/
JSM.Ob013e318204lde0. PubMed PMID: 21358500.

© 2014 American Academy of Orthopaedic Surgeons 2014 Shoulder and Elbow Self-Assessment Examination
2014 Shoulder and Elbow Self-Assessment Examination Answer Book• 71

CLINICAL SITUATION FOR QUESTIONS 70 THROUGH 72


A 17-year-old high school football player sustains a traumatic anterior shoulder dislocation resulting in
a small bony Bankart lesion and small Hill-Sachs lesion. The patient undergoes an arthroscopic Bankart
repair with incorporation of the bone fragment and returns to play football the following year. He has
a recurrent dislocation at football practice, but decides to finish the football season before considering
additional treatment. He sustains 9 additional dislocations, with the last dislocation occurring while
sleeping.

Question 70
What diagnostic test is best when planning revision surgery?

1. CT scan with 3-dimensional (3-D) reconstructions


2. Ultrasound
3. MRI scan
4. Arthrogram

PREFERRED RESPONSE: 1

Question 71
The patient has eroded one-third of the inferior glenoid surface area. What is the most appropriate
treatment?

1. Revision arthroscopic Bankart repair with capsular shift


2. Open Bankart repair with capsular shift
3. Repair of infraspinatus tendon into the Hill-Sachs defect (remplissage procedure)
4. Coracoid transfer to the glenoid (Latarjet procedure)

PREFERRED RESPONSE: 4

Question 72
Which patients are clinically most dissatisfied after revision instability surgery?

1. Patients with pain before surgery


2. Patients younger than 25 years of age
3. Patients older than 55 years of age
4. Recreational athletes

PREFERRED RESPONSE: 1

<C 2014 American Academy of Orthopaedic Surgeons 2014 Shoulder and Elbow Self-Assessment Examination
72 • American Academy of Orthopaedic Surgeons

DISCUSSION FOR QUESTIONS 70 THROUGH 72


A failed bony Bankart repair with multiple dislocations can further erode the anteroinferior glenoid,
changing the sagittal morphology of the glenoid into an "inverted pear." Quantitative bone loss is best
evaluated by CT scan with 3-D reconstructions and subtraction of the humeral head. :MRI and ultrasound
can assist in evaluating soft-tissue injury, but they are not as helpful in determining bone loss compared
to a CT scan. An arthrogram alone is not sufficient to evaluate bone loss. Bone loss exceeding 30%
necessitates glenoid augmentation with either a Latarjet procedure or iliac crest bone grafting. A revision
arthroscopic or open Bankart repair with capsular shift or remplissage do not address bone loss. The
Latarjet procedure can effectively restore stability with glenoid bone loss and after failed stabilizing
procedures. Patients with pain before surgery are more likely to have pain after surgery. Age and activity
level are lesser influences on satisfaction.

RECOMMENDED READINGS FOR QUESTIONS 70 THROUGH 72


Schmid SL, Farshad M, Catanzaro S, Gerber C. The Latarjet procedure for the treatment ofrecurrence
of anterior instability of the shoulder after operative repair: a retrospective case series of forty-nine
consecutive patients. J Bone Joint SurgAm. 2012 Jun 6;94(11):e75. doi: 10.2106/JBJS.K.00380. PubMed
PMID: 22637215.

Piasecki DP, Verma NN, Romeo AA, Levine WN, Bach BR Jr, Provencher MT. Glenoid bone deficiency
in recurrent anterior shoulder instability: diagnosis and management. J Am Acad Orthop Surg. 2009
Aug;l7(8):482-93. Review. PubMed PMID: 19652030.

Burkhart SS, De Beer JF. Traumatic glenohumeral bone defects and their relationship to failure of
arthroscopic Bankart repairs: significance of the inverted-pear glenoid and the humeral engaging Hill-
Sachs lesion. Arthroscopy. 2000 Oct;l6(7):677-94.PubMed PMID: 11027751.

END OF SERIES

© 2014 American Academy of Orthopaedic Surgeons 2014 Shoulder and Elbow Self-Assessment Examination
2014 Shoulder and Elbow Self-Assessment Examination Answer Book• 73

Figure 73

Question 73
Figure 73 is the radiograph of a 78-year-old man who has had 8 months of gradually progressive
right shoulder pain. He temporarily responds to a corticosteroid injection administered by his primary
physician, but his symptoms quickly return. He reports significant interference with activities of daily
living and recreational activities. Examination demonstrates active range of motion to 90 degrees' forward
elevation, 20 degrees' external rotation at the side, and 50 degrees' in the abducted position, with pain at
end range. The most appropriate next treatment step is

I. Arthroscopic glenohumeral debridement, synovectomy, and biceps tenotomy


2. Total shoulder arthroplasty (TSA)
3. Reverse TSA (rTSA)
4. Humeral head arthroplasty without glenoid resurfacing

PREFERRED RESPONSE: 3

DISCUSSION
This patient presents with a clinical history, examination, and imaging consistent with end-stage rotator
cuff tear arthropathy. Recommended treatment is rTSA. With significantly limited function and advanced
radiographic changes, arthroscopic intervention is unlikely to provide significant clinical benefit. TSA,
with or without rotator cuff repair, likely would lead to early mechanical failure of the glenoid component
(edge loading, or "rocking horse" glenoid). Hemiarthroplasty was previously the recommended treatment
option, prior to the reintroduction of the reverse implant. However, current data suggest reverse
arthroplasty provides a more predictable outcome (pain relief, improved function) and less need for
surgical revision.

© 2014 American Academy of Orthopaedic Surgeons 2014 Shoulder and Elbow Self-Assessment Examination
74 • American Academy of Orthopaedic Surgeons

RECOJ\1MENDED READINGS
Cuff D, Pupello D, Virani N, Levy J, Frankle M. Reverse shoulder arthroplasty for the treatment ofrotator
cuff deficiency. J Bone Joint SurgAm. 2008 Jun;90(6):1244-51. doi: 10.2106/JBJS.G.00775. PubMed
PMID: 18519317.

Drake GN, O'Connor DP, Edwards TB. Indications for reverse total shoulder arthroplasty in rotator cuff
disease. Clin Orthop Relat Res. 2010 Jun;468(6):1526-33. doi: 10.1007/s11999-009-1188-9. Review.
PubMed PMID: 20049573.

Leung B, Horodyski M, StrukAM, Wright TW. Functional outcome ofhemiarthroplasty compared with
reverse total shoulder arthroplasty in the treatment of rotator cuff tear arthropathy. J Shoulder Elbow Surg.
2012 Mar;21(3):319-23. doi: 10.1016/j.jse.2011.05.023. Epub 2011 Aug 26. PubMed PMID: 21872496.

Figure 74a Figure 74b

CLINICAL SITUATION FOR QUESTIONS 74 AND 75


Figures 74a and 74b are the radiographs of a 20-year-old collegiate football player who has had recurring
episodes of right shoulder instability after undergoing an arthroscopic capsulolabral repair 1 year ago.
Clinically, he demonstrates a positive anterior apprehension test.

Question 74
What is the most appropriate diagnostic test?

1. MRI scan
2. Ultrasound
3. CT Scan
4. Electromyogram and nerve conduction study

PREFERRED RESPONSE: 3

© 2014 American Academy of Orthopaedic Surgeons 2014 Shoulder and Elbow Self-Assessment Examination
2014 Shoulder and Elbow Self-Assessment Examination Answer Book• 75

Question 75
If nonsurgical treatment has failed, what surgical procedure will best reduce the risk for recurrent
instability?

1. Diagnostic shoulder arthroscopy with labral repair


2. Diagnostic shoulder arthroscopy with open capsular shift
3. Diagnostic shoulder arthroscopy with coracoid transfer
4. Diagnostic shoulder arthroscopy with thermal capsulorrhaphy

PREFERRED RESPONSE: 3

DISCUSSION FOR QUESTIONS 74 AND 75


If nonsurgical treatment has failed in the revision setting, the amount of bone loss should be quantified.
The current standard for quantification of glenoid bone loss is CT scan with or without digital subtraction
of the humeral head. An initial diagnostic arthroscopy may permit calculation of glenoid bone loss. The
glenoid bare-spot method popularized by Lo and associates provides a reliable estimate of bone loss.
Percentage of bone loss is calculated by subtracting the distance from the anterior rim to the bare spot
from the posterior rim-bare spot distance divided by twice the posterior rim-bare spot distance. The critical
limits of glenoid bone loss are based on a combination of cadaveric and clinical reports. Nonsurgical
management may still be a reasonable choice with less than 20% glenoid bone loss in low-demand
individuals, patients with high surgical risk secondary to medical comorbidities, and voluntary dislocators.
When addressing recurrent anterior instability of the shoulder, it is imperative to assess both soft-tissue
and bone injury. Particular attention must be paid to glenoid and humeral head deficiencies. Patient-
specific demands should be considered when discussing treatment options. In a high-demand patient such
as this contact athlete, surgical treatment is appropriate. In general, if glenoid bone loss is less than 15%,
a soft-tissue stabilization procedure may be all that is necessary. In those with 15% to 25% bone loss,
arthroscopic stabilization with bone fragment incorporation may be performed if local bone is available.
In the setting of a high-demand patient with no local bone for repair, coracoid transfer, iliac crest bone
autograft, or distal tibial allograft is appropriate. With more than 25% bone loss, the glenoid deficiency
must be addressed. In this scenario, the athlete demonstrates more than 25% bone loss involving the
anteroinferior glenoid. This deficiency must be addressed to restore stability to the glenohumeral joint. In
a high-demand patient (contact athlete), augmentation with iliac crest bone graft, distal tibial allograft, or a
coracoid transfer procedure is appropriate if local bone is not available.

IC 2014 American Academy of Orthopaedic Surgeons 2014 Shoulder and Elbow Self-Assessment Examination
76 • American Academy of Orthopaedic Surgeons

RECOMMENDED READINGS FOR QUESTIONS 74 AND 75


Boileau P, Villalba M, Hery JY, Baig F, Ahrens P, Neyton L. Risk factors for recurrence of shoulder
instability after arthroscopic Bankart repair. J Bone Joint Surg Am. 2006 Aug;88(8):1755-63. PubMed
PMID: 16882898.

Provencher MT, Bhatia S, Ghodadra NS, Grumet RC, Bach BR Jr, Dewing CB, LeClere L, Romeo AA.
Recurrent shoulder instability: current concepts for evaluation and management of glenoid bone loss. J
Bone Joint SurgAm. 2010 Dec;92 Suppl 2:133-51. Review. PubMed PMID: 21123597.

Piasecki DP, Verma NN, Romeo AA, Levine WN, Bach BR Jr, Provencher MT. Glenoid bone deficiency
in recurrent anterior shoulder instability: diagnosis and management. J Am Acad Orthop Surg. 2009
Aug;l7(8):482-93. Review. PubMed PMID: 19652030.

Hovelius L, Sandstrom B, Olofsson A, Svensson 0, Rahme H. The effect of capsular repair, bone block
healing, and position on the results of the Bristow-Latarjet procedure (study III): long-term follow-up
in 319 shoulders. J Shoulder Elbow Surg. 2012 May;21(5):647-60. Epub 2011 Jun 29. PubMed PMID:
21719316.

END OF SERIES

Question 76
A 33-year old man sustains a posterior elbow dislocation after a fall.Attempts at closed reduction result
in recurrent instability. What is the most common ligamentous injury found at the time of surgical
stabilization?

1. Midsubstance tear of the lateral ulnar collateral ligament


2. Proximal avulsion of the ulnar collateral ligament
3. Proximal avulsion of the lateral ulnar collateral ligament
4. Distal bony avulsion of the ulnar collateral ligament from the sublime tubercle

PREFERRED RESPONSE: 3

DISCUSSION
Classic posterior elbow dislocations result from a posterolateral rotatory mechanism, whereby the hand is
fixed (typically on the ground) while the weight of the body creates a valgus and external rotation moment
on the elbow. This results first in tearing of the lateral collateral ligament that proceeds medially through
the anterior and posterior joint capsules, ending with potential involvement of the ulnar collateral ligament
(but this is not universal). McKee and associates assessed the lateral soft-tissue injury pattern of elbow
dislocations with and without associated fractures at the time of surgery. Injury to the lateral collateral
ligament complex was seen in every case, with avulsion from the distal humerus as the most common
finding. Midsubstance tears, proximal avulsions, and distal bony avulsions of the ulnar collateral ligament
are less common.

© 20I4 American Academy of Orthopaedic Surgeons 2014 Shoulder and Elbow Self-Assessment Examination
2014 Shoulder and Elbow Self-Assessment Examination Answer Book • 77

RECOMMENDED READINGS
McKee MD, Schemitsch EH, Sala MJ, O'driscoll SW. The pathoanatomy of lateral ligamentous disruption
in complex elbow instability. J Shoulder Elbow Surg. 2003 Jul-Aug;12(4):391-6. PubMed PMID:
12934037.

O'Driscoll SW, Morrey BF, Korinek S, An KN. Elbow subluxation and dislocation. A spectrum of
instability. Clin Orthop Relat Res. 1992 Jul;(280):186-97. PubMed PMID: 1611741.

Question 77
A 25-year-old man is planning to have an elbow contracture release. His elbow range of motion is 40
degrees to 90 degrees offlexion. He has no heterotopic ossification. His ring and small fingers become
numb as his elbow approaches his flexion endpoint. There is no evidence of instability of the ulna-humeral
or radioulnar joints. To achieve the best possible outcome, the surgeon should

1. include postsurgical elbow continuous passive motion (CPM).


2. perform the surgery open.
3. decompress the ulnar nerve.
4. release the anterior band of the medial collateral ligament.

PREFERRED RESPONSE: 3

DISCUSSION
The patient is exhibiting signs of ulnar neuropathy. The surgeon should be sure to decompress and
possibly transpose the ulnar nerve, if unstable, to prevent worsening neuropathy after surgery. CPM has
not been shown to be of benefit after contracture release. Equal success rates have been shown for open
and arthroscopic contracture releases. The anterior band of the medial collateral ligament is important
to maintain valgus stability of the elbow. The posterior band can be released to improve flexion without
increasing concern for elbow instability.

RECOMMENDED READINGS
Charalambous CP, Morrey BF. Posttraumatic elbow stiffness. J Bone Joint SurgAm. 2012 Aug
1;94(15):1428-37. doi: 10.2106/JBJS.K.00711. Review. PubMed PMID: 22854997.

Williams BG, Sotereanos DG, Baratz ME, Jarrett CD, Venouziou AI, Miller MC. The contracted elbow: is
ulnar nerve release necessary? J Shoulder Elbow Surg. 2012 Jun 26. [Epub ahead of print] PubMed PMID:
22743068.

Lindenhovius AL, van de Luijtgaarden K, Ring D, Jupiter J. Open elbow contracture release:
postoperative management with and without continuous passive motion. J Hand Surg Am. 2009 May-
Jun;34(5):858-65. Epub 2009 Apr 11. PubMed PMID: 19362791.

© 2014 American Academy of Orthopaedic Surgeons 2014 Shoulder and Elbow Self-Assessment Examination
78 • American Academy of Orthopaedic Surgeons

Figure 78a Figure 78b

Question 78
Figures 78a and 78b are the radiographs of a 47-year-old right-hand-dominant woman who has a 3-month
history of gradually progressive right shoulder pain. She reports no previous trauma, but does report pain
at night and with activity such as weight training. Examination demonstrates active and passive range of
motion to be 110 degrees forward elevation, external rotation to 20 degrees, and internal rotation to the
sacrum. The next treatment step should include

1. an l\1RI of the shoulder.


2. a physical therapy referral for rotator cuff strengthening and proprioceptive exercise.
3. a home stretching program and corticosteroid injection.
4. arthroscopic glenohumeral capsular release.

PREFERRED RESPONSE: 3

DISCUSSION
This patient has idiopathic adhesive glenohumeral stiffness. Most patients with this condition are women
between 40 and 60 years of age with no specific mechanism of onset. Patients typically develop pain, at
which point the disease is marked by significant inflammation. This patient is likely in the second stage
of the disease, marked by inflammation and early fibrosis of the joint capsule, leading to joint stiffness.
Associated conditions include diabetes mellitus and hypothyroidism, although there is no explainable
cause for most cases. The most appropriate treatment step at this stage is an intra-articular glenohumeral
corticosteroid injection, most often in conjunction with either a supervised or home-based capsular
stretching program. Physical therapy that prioritizes toward rotator cuff strengthening is more appropriate
for patients with isolated subacromial impingement syndrome and may worsen symptoms in patients
with stiff shoulders. An l\1RI scan likely would not alter initial treatment for patients who are stiff at
presentation. Arthroscopic glenohumeral capsular release is reserved for those patients who fail initial
attempts at nonsurgical management and remain functionally limited.

© 2014 American Academy of Orthopaedic Surgeons 2014 Shoulder and Elbow Self-Assessment Examination
2014 Shoulder and Elbow Self-Assessment Examination Answer Book • 79

RECOMMENDED READINGS
Marx RG, Malizia RW, Kenter K, Wickiewicz TL, Hannafin JA. Intra-articular corticosteroid injection for
the treatment of idiopathic adhesive capsulitis of the shoulder. HSS J. 2007 Sep;3(2):202-7. doi: 10.1007/
s11420-007-9044-5. PubMed PMID: 18751795.

Johnson TS, Mesfin A, Farmer KW, McGuigan LA, Alamo JG, Jones LC, Johnson DC. Accuracy of
intra-articular glenohumeral injections: the anterosuperior technique with arthroscopic documentation.
Arthroscopy. 2011 Jun;27(6):745-9. doi: 10.1016/j.arthro.2011.02.010. PubMed PMID: 21624668.

Shin SJ, Lee SY. Efficacies of corticosteroid injection at different sites of the shoulder for the treatment
of adhesive capsulitis. J Shoulder Elbow Surg. 2012 Sep 20. doi:pii: S1058-2746(12)00278-9. 10.1016/j.
jse.2012.06.015. [Epub ahead of print] PubMed PMID: 22999847.

Neviaser AS, Hannafin JA. Adhesive capsulitis: a review of current treatment. Am J Sports Med. 2010
Nov;38(11):2346-56. doi: 10.1177/0363546509348048. Epub 2010 Jan 28. Review. PubMed PMID:
20110457.

RESPONSES FOR QUESTIONS 79 THROUGH 82


1. Rotator cuff and scapular stabilizer strengthening exercises
2. Diagnostic and therapeutic corticosteroid injection
3. Arthroscopic debridement
4. Completion ofrotator cuff tear, repair, and biceps tenotomy
5. Acromioplasty
6. Repair of rotator cuff and superior labrum anterior to posterior (SLAP) repair
7. Repair of subscapularis tendon and biceps tenodesis

Figure 79

Question 79
What is contraindicated in a patient with a partial articular supraspinatus tendon avulsion lesion and the
axial MRI scan shown in Figure 79?

PREFERRED RESPONSE: 5
© 2014 American Academy of Orthopaedic Surgeons 2014 Shoulder and Elbow Self-Assessment Examination
80 • American Academy of Orthopaedic Surgeons

Question 80
What is the most appropriate initial treatment in a 25-year-old professional baseball player with a partial-
thickness rotator cuff tear involving 40% thickness of the tendon?

PREFERRED RESPONSE: 1

Figure 81

Question 81
What is the most appropriate definitive treatment in a 65-year-old man who has experienced symptoms for
more than 1 year and has a partial-thickness rotator cuff tear involving 90% of the tendon and arthroscopy
shown in Figure 81?

PREFERRED RESPONSE: 4

© 2014 American Academy of Orthopaedic Surgeons 2014 Shoulder and Elbow Self-Assessment Examjnation
2014 Shoulder and Elbow Self-Assessment Examination Answer Book• 81

Figure 82

Question 82
What is the most appropriate treatment for a 25-year-old man 1 week after falling off a ladder? His axial
T2-weighted J\.1RI scan is shown in Figure 82.

PREFERRED RESPONSE: 7

DISCUSSION FOR QUESTIONS 79 THROUGH 82


Acromioplasty can destabilize an os acromiale and is contraindicated. Initial treatment of partial-thickness
rotator cuff tears should be nonsurgical, with a focus on rehabilitative exercises. Stiffness is more
common after rotator cuff repair with concomitant SLAP repair, and SLAP repair is not advocated in most
people older than 40 years of age. Rotator cuff repair with biceps tenotomy or tenodesis is preferred to a
SLAP repair in this patient. Figure 51-4 shows a complete tear of the subscapularis tendon with medial
subluxation of the biceps tendon. In young patients, acute repair is preferred with stabilization of the
biceps tendon.

RECO11MENDED READINGS FOR QUESTIONS 79 THROUGH 82


Oh JH, Kim SH, Kwak SH, Oh CH, Gong HS. Results of concomitant rotator cuff and SLAP repair are
not affected by unhealed SLAP lesion. J Shoulder Elbow Surg. 2011 Jan;20(1):138-45. doi: 10.1016/j.
jse.2010.04.003. Epub 2010 Jul 15. PubMed PMID: 20634099.

Wolff AB, Sethi P, Sutton KM, Covey AS, Magit DP, Medvecky M. Partial-thickness rotator cuff tears. J
AmAcad Orthop Surg. 2006 Dec;14(13):715-25. PubMed PMID: 17148619.

END OF SERIES

© 2014 American Academy of Orthopaedic Surgeons 2014 Shoulder and Elbow Self-Assessment Examination
82 • American Academy of Orthopaedic Surgeons

Figure 83a Figure 83b

Question 83
Figures 83a and 83b are the radiographs of a 53-year-old otherwise healthy homemaker who had a
syncopal episode and sustained a ground-level fall and injury to her right elbow. She presently admits to
right elbow pain, swelling, and an inability to bend her elbow. What is the best initial treatment for this
injury?

1. Closed reduction with immobilization


2. Closed reduction with percutaneous pinning
3. Open reduction, bicolumnar fixation with plate and screws
4. Open reduction, bicolumnar fixation with Kirschner wires

PREFERRED RESPONSE: 3

DISCUSSION
The radiographs and CT scans indicate a comminuted and displaced intra-articular fracture of the distal
humerus. Rigid internal fixation with bicolumnar orthogonal or parallel plating is the treatment of choice
for most fractures of the distal humerus that involve the joint surface. Closed reduction and variations
thereof will not yield a stable environment for healing. To achieve adequate exposure for fixation, a
chevron olecranon osteotomy is the preferred approach. Disadvantages associated with this approach
include complications such as nonunion of the osteotomy site and intra-articular adhesions. Prominent
hardware may need to be removed during a secondary procedure, and intraoperative conversion to an
elbow arthroplasty may be limited. The most common complications after open reduction and internal
fixation include elbow stiffness, nonunion (2%-10%), and ulnarneuropathy (0%-12%).

© 2014 American Academy of Orthopaedic Surgeons 2014 Shoulder and Elbow Self-Assessment Examination
2014 Shoulder and Elbow Self-Assessment Examination Answer Book• 83

RECO:rv1MENDED READINGS
Galano GJ, Ahmad CS, Levine WN. Current treatment strategies for bicolumnar distal humerus fractures.
J AmAcad Orthop Surg. 2010 Jan;18(1):20-30. Review. PubMed PMID: 20044489.

Ring D, Gulotta L, Chin K, Jupiter JB. Olecranon osteotomy for exposure of fractures and nonunions of
the distal humerus. J Orthop Trauma. 2004 Aug;18(7):446-9. PubMed PMID: 15289692.

Sanchez-Sotelo, J. Open reduction and internal fixation of supracondylar and intercondylar fractures. In:
Wiesel SW, ed. Operative Techniques in Orthopaedic Surgery. Philadelphia, PA: Lippincott Williams &
Wilkins; 2011:3329-3336.

Figure 84

CLINICAL SITUATION FOR QUESTIONS 84 THROUGH 87


Figure 84 is the glenoid CT scan of a 20-year-old man who dislocated his shoulder anteriorly while
playing football. He had persistent instability 2 months after the injury, but he did not have a sulcus sign
or posterior instability. He underwent an arthroscopic Bankart repair with 4 bioabsorbable anchors with
simple sutures through the labrum and capsule. He did not have an engaging Hill-Sachs lesion, the rotator
cuff was unremarkable, and the capsule was not tom from the humerus. After surgery, he did well for 6
months until he jumped into a lake and again dislocated his shoulder anteriorly. He says his shoulder no
longer felt stable after his reduction.

Question 84
What is the most likely reason this patient's arthroscopic Bankart repair failed?

1. The surgeon did not use enough anchors to repair the labrum.
2. The surgeon did not recognize significant bone loss of the anterior glenoid.
3. The patient returned to full activity too soon.
4. The patient has unrecognized multidirectional instability.

PREFERRED RESPONSE: 2
© 2014 American Academy of Orthopaedic Surgeons 2014 Shoulder and Elbow Self-Assessment Examination
84 • American Academy of Orthopaedic Surgeons

Question 85
This patient would like to return to football and perform normal activities of daily living without worrying
about another dislocation. What treatment would you recommend?

1. Open Bankart repair


2. Coracoid transfer
3. Revision arthroscopic labrum repair
4. Arthroscopic pan capsular plication and labrum repair

PREFERRED RESPONSE: 2

Question 86
What is the most common early complication of the revision procedure for this patient?

1. Loss of external rotation


2. Loss of internal rotation
3. Recurrent instability
4. Subscapularis tear

PREFERRED RESPONSE: 1

Question 87
What is the most common late complication of the revision procedure for this patient?

1. Glenohumeral arthritis
2. Bone graft absorption
3. Anterior ligament attenuation
4. Rotator cuff tear

PREFERRED RESPONSE: 1

© 2014 American Academy of Orthopaedic Surgeons 20 I4 Shoulder and Elbow Self-Assessment Examination
2014 Shoulder and Elbow Self-Assessment Examination Answer Book • 85

DISCUSSION FOR QUESTIONS 84 THROUGH 87


The CT scan shows bone loss exceeding 20% on the anterior glenoid, which is the most likely reason
the arthroscopic Bankart repair failed. One study showed that using 3 or fewer anchors increases risk for
failure; 4 anchors were used in this patient, so that is not the likely cause of failure. The patient returned to
full activity 6 months after surgery, which is the usual time needed to regain full strength in the shoulder
and ensure complete labrum healing. Suture configuration has not been shown to affect failure rates. The
patient did not have signs of multidirectional instability such as a sulcus sign on examination, instability
without a labral tear, or excessive translation of the humeral head posteriorly on examination. This patient
has recurrent instability due to glenoid bone loss, so the procedure of choice would need to restore the
anterior bone to the glenoid. The coracoid transfer procedure uses the coracoid for bone restoration,
but iliac crest bone graft would be appropriate as well. An open Bankart repair, arthroscopic capsular
plication, or a revision arthroscopic repair are all soft-tissue procedures, which do not correct the bone
loss. Braces may work to allow a patient to finish a season before having surgery, but will not allow a
return to activities of daily living without instability. The most common complication of the coracoid
transfer is a loss of external rotation. The rate of recurrent instability is low. Most patients regain all of
their internal rotation. The technique for the coracoid transfer splits the subscapularis muscle, so a tear
of the muscle is rare. Axillary or musculocutaneous nerve palsies are rare after this procedure, but can
occur if the nerves are not protected and mobilized during the dissection of the conjoint tendon. The most
common long-term complication is early arthritis of the glenohumeral joint. Most cases of arthritis are
asymptomatic and appear on follow-up radiographs. The graft rarely absorbs, and tears of the rotator cuff
are uncommon with this procedure and infection is rare. Anterior ligament attenuation is uncommon, and
some surgeons do not even repair the anterior labrum or capsule because this can lead to a loss of external
rotation after surgery.

© 2014 American Academy of Orthopaedic Surgeons 2014 Shoulder and Elbow Self-Assessment Examination
86 • American Academy of Orthopaedic Surgeons

RECOMMENDED READINGS FOR QUESTIONS 84 THROUGH 87


Burkhart SS, De Beer JF. Traumatic glenohumeral bone defects and their relationship to failure of
arthroscopic Ban.kart repairs: significance of the inverted-pear glenoid and the humeral engaging Hill-
Sachs lesion. Arthroscopy. 2000 Oct;l6(7):677-94. PubMed PMID: 11027751.

Ochoa E Jr, Burkhart SS. Glenohumeral bone defects in the treatment of anterior shoulder instability. Instr
Course Leet. 2009;58:323-36. PubMed PMID: 19385546.

Schmid SL, Farshad M, Catanzaro S, Gerber C. The Latarjet procedure for the treatment ofrecurrence
of anterior instability of the shoulder after operative repair: a retrospective case series of forty-nine
consecutive patients. J Bone Joint SurgAm. 2012 Jun 6;94(11):e75. doi: 10.2106/JBJS.K.00380. PubMed
PMID: 22637215.

Neyton L, Young A, Dawidziak B, Visona E, Hager JP, Fournier Y, Walch G. Surgical treatment of
anterior instability in rugby union players: clinical and radiographic results of the Latarjet-Patte procedure
with minimum 5-year follow-up. J Shoulder Elbow Surg. 2012 Dec;21(12):1721-7. doi: 10.1016/j.
jse.2012.01.023. Epub 2012 May 5. PubMed PMID: 22565042.

Hovelius L, Vikerfors 0, Olofsson A, Svensson 0, Rahme H. Bristow-Latarjet and Bankart: a comparative


study of shoulder stabilization in 185 shoulders during a seventeen-year follow-up. J Shoulder Elbow
Surg. 2011 Oct;20(7):1095-101. doi: 10.1016/j.jse.2011.02.005. Epub 2011 May 24. PubMed PMID:
21602067.

Hovelius L, Saeboe M. Neer Award 2008: Arthropathy after primary anterior shoulder dislocation--223
shoulders prospectively followed up for twenty-five years. J Shoulder Elbow Surg. 2009 May-
Jun;18(3):339-47. doi: 10.1016/j.jse.2008.11.004. Epub 2009 Feb 28. PubMed PMID: 19254851.

END OF SERIES

Question 88
Complete transection of the ulnar nerve at the elbow will result in

1. loss of sensation on the ulnar side of the index finger.


2. weakness with thumb extension.
3. weakness with elbow flexion.
4. weakness with finger abduction.

PREFERRED RESPONSE: 4

DISCUSSION
Ulnar nerve lesions manifest with weakness in the finger abductor muscles. There will be loss of interossei
muscle function as well as the third and fourth lumbricals. Extensor pollicis longus function is based on
the posterior interosseous nerve (radial), not the ulnar. The index finger has sensation from the median
nerve distribution. Elbow flexion strength is not dependent on the ulnar nerve.

© 2014 American Academy of Orthopaedic Surgeons 2014 Shoulder and Elbow Self-Assessment Examination
2014 Shoulder and Elbow Self-Assessment Examination Answer Book• 87

RECOMMENDED READINGS
Hoppenfeld S. Physical Examination of the Spine and Extremities. New York: Appleton-Century-Crofts;
1976:93-104.

Netter FH. The CIBA Collection of Medical Illustrations. Musculoskeletal System Part I. Vol 8. Summit,
NJ: Ciba-Geigy Corporation; 1987:28.

Figure 89a Figure 89b

Question 89
Figures 89a and 89b are the radiograph and MRJ scan of a 40-year-old man who fell down a flight of
stairs. His upper arm is bruised and painful, and global weakness in the shoulder girdle function is noted.
A radiograph is ordered to rule out a fracture or dislocation. You should recommend

1. immediate open reduction and internal fixation of the fracture.


2. closed treatment with serial radiographs.
3. fracture fragment excision and deltoid repair.
4. rest, ice, anti-inflammatory medications, and a home exercise program.

PREFERRED RESPONSE: 4

DISCUSSION
The patient has an os acromiale. The type shown is of the meso-acromion. This is not an acute fracture;
well corticated ends are seen on the axillary radiograph and there is no bone edema on the T2 axial
MRJ image. A trial of nonsurgical care that includes rest, ice, and anti-inflammatory medication is
recommended. If a patient continues to have symptoms, an arthroscopic evaluation is needed to
determine if the os is mobile and if os fixation is appropriate.

© 2014 American Academy of Orthopaedic Surgeons 2014 Shoulder and Elbow Self-Assessment Examination
88 • American Academy of Orthopaedic Surgeons

RECOMMENDED READINGS
Sammarco VJ. Os acromiale: frequency, anatomy, and clinical implications. J Bone Joint Surg Am. 2000
Mar;82(3):394-400. PubMed PMID: 10724231.

Harris JD, Griesser MJ, Jones GL. Systematic review of the surgical treatment for symptomatic os
acromiale. Int J Shoulder Surg. 2011 Jan;5(1):9-16. PubMed PMID: 21660192.

Abboud JA, Silverberg D, Pepe M, Beredjiklian PK, Iannotti JP, Williams GR, Ramsey ML. Surgical
treatment of os acromiale with and without associated rotator cuff tears. J Shoulder Elbow Surg. 2006
May-Jun;15(3):265-70. PubMed PMID: 16679224.

Figure 90

Question 90
Figure 90 is the initial radiograph of a 28-year-old woman who sustained an acute right elbow injury.
Following closed treatment under sedation in the emergency department, the elbow is seen to be stable
through an arc from full flexion down to 30 degrees short of full extension, while the forearm is pronated
but only to 75 degrees short of full extension while in supination. What structure is most likely to remain
intact?

1. Lateral ulnar collateral ligament


2. Radial head
3. Posterior band of the medial collateral ligament (MCL)
4. Anterior band of the MCL

PREFERRED RESPONSE: 4

© 2014 American Academy of Orthopaedic Surgeons 2014 Shoulder and Elbow Self-Assessment Examination
2014 Shoulder and Elbow Self-Assessment Examination Answer Book• 89

DISCUSSION:
The most common pattern of elbow dislocation is associated with posterolateral rotatory instability. This
pattern begins with valgus, axial load, and supination rotating the radial head posterior with respect to
the capitellum and failure of the lateral ulnar collateral ligament. The posterior band of the MCL tears
next, and the anterior band of the MCL is last to fail. In elbows with an intact anterior band of the MCL,
forearm pronation will place this structure under tension and assist in maintaining joint reduction. If
this band is tom, pronation will lead to medial joint space widening. Radial head fractures, along with
coronoid fractures, are common associated injuries, as seen in the radiographs for this patient.

RECOMMENDED READINGS
O'Driscoll SW, Morrey BF, Korinek S, An KN. Elbow subluxation and dislocation. A spectrum of
instability. Clin Orthop Relat Res. 1992 Ju1;(280):186-97. PubMed PMID: 1611741.

Olsen BS, S0jbjerg JO, Nielsen KK, Vaesel MT, Dalstra M, Sneppen 0. Posterolateral elbow joint
instability: the basic kinematics. J Shoulder Elbow Surg. 1998 Jan-Feb;7(1):19-29. PubMed PMID:
9524337.

O'Driscoll SW. Classification and evaluation of recurrent instability of the elbow. Clin Orthop Relat Res.
2000 Jan;(370):34-43. Review. PubMed PMID: 10660700.

© 2014 American Academy of Orthopaedic Surgeons 2014 Shoulder and Elbow Self-Assessment Examination
90 • American Academy of Orthopaedic Surgeons

Figure 91a Figure 91b Figure 91c Figure 91d

Question 91
Figures 91a through 91d are the radiographs of an 86-year-old man who lives independently who has
fallen down the stairs. He has an isolated elbow injury. What treatment option is most likely to offer the
most rapid return of function and pain relief?

I. Open reduction and internal :fixation (ORIF)


2. Cast treatment for 4 weeks followed by static splinting
3. Percutaneous pinning
4. Total elbow arthroplasty (TEA)

PREFERRED RESPONSE: 4

DISCUSSION
Advantages of TEA for fracture in elderly patients include preservation of the extensor mechanism, early
mobilization, and avoiding complications of fixation; however, there are lifetime activity limitations
and risk for loosening over time. Reoperation rates may be lower, functional outcomes improved, and
results more predictable for TEA than ORIF in elderly populations. Cast treatment leads to unacceptable
rates of stiffness and disability. Percutaneous :fixation does not provide rigid enough fixation in the adult
population.

© 2014 American Academy of Orthopaedic Surgeons 2014 Shoulder and Elbow Self-Assessment Examination
2014 Shoulder and Elbow Self-Assessment Examination Answer Book • 91

RECOMMENDED READINGS
Sanchez-Sotelo J. Fractures of the distal humerus. In: Galatz L, ed. Orthopaedic Knowledge Update
Shoulder and Elbow 3. Rosemont, IL: American Academy of Orthopaedic Surgeons; 2008:507-516.

Sanchez-Sotelo J. Distal humeral fractures: role of internal fixation and elbow arthroplasty. Instr Course
Leet. 2012;61:203-13. PubMed PMID: 22301233.

McKee MD, Veillette CJ, Hall JA, Schemitsch EH, Wild LM, McCormack R, Perey B, Goetz T, Zomar
M, Moon K, Mandel S, Petit S, Guy P, Leung I. A multicenter, prospective, randomized, controlled trial
of open reduction--internal fixation versus total elbow arthroplasty for displaced intra-articular distal
humeral fractures in elderly patients. J Shoulder Elbow Surg. 2009 Jan-Feb;18(1):3-12. doi: 10.1016/j.
jse.2008.06.005. Epub 2008 Sep 26. PubMed PMID: 18823799.

Kim JM, Mudgal CS, Konopka JF, Jupiter JB. Complications of total elbow arthroplasty. J Am Acad
Orthop Surg. 2011 Jun;19(6):328-39. Review. PubMed PMID: 21628644.

Question 92
A 68-year-old right-hand-dominant man underwent a right total shoulder arthroplasty (TSA) 3 months
ago. He was started on passive range of motion and started active motion 6 weeks after surgery. He notes
that he fell onto his outstretched right arm 2 weeks ago but did not seek care. His primary symptom is
poor active elevation of the right shoulder. His right shoulder motion has active elevation of 45 degrees,
passive elevation of 140 degrees, 95-degree external rotation, and internal rotation to L3. His left shoulder
has active and passive elevation of 160 degrees, external rotation of 70 degrees, and internal rotation to
T12. The right shoulder radiographs show a concentric total shoulder arthroplasty with no fractures or
other abnormalities. What is the most appropriate treatment at this point?

1.Reassurance and a review of his rehabilitation program with an emphasis on deltoid


strengthening
2. Open repair of the subscapularis tendon
4. Latissimus dorsi tendon transfer
5. Revision to reverse TSA

PREFERRED RESPONSE: 2

<C 2014 American Academy of Orthopaedic Surgeons 2014 Shoulder and Elbow Self-Assessment Examination
92 • American Academy of Orthopaedic Surgeons

DISCUSSION
This patient had a fall approximately 2½ months following a TSA. He now has poor active elevation but
good passive motion and external rotation exceeding that of the contralateral shoulder. Rupture of the
subscapularis tendon, which would have been released and repaired intrasurgically, would be the primary
concern in this scenario. In the native shoulder treated surgically for instability, subscapularis failure
can produce pain, weakness of abdominal press and lumbar pushoff, apprehension, and frank instability.
Further delay in treatment of the tendon failure with therapy is not indicated because this will lead to
further muscle atrophy and adhesions to the scapula and overlying brachia! plexus. Augmentation or
replacement with a transfer of the superior portion of the pectoralis major muscle is sometimes required.
Transfer of the pectoralis minor muscle is also described. However, latissimus dorsi transfer is described
for irreparable supraspinatus deficiency. Revision to a reverse TSA can be considered as a salvage of
a persistently unstable shoulder, but will not be the primary treatment for this shoulder if radiograph
findings are normal.

RECOMMENDED READINGS
Lazarus MD, Harryman DT II. Open repairs for anterior instability. In: Warner J, Iannotti J, Gerver R, eds.
Complex and Revision Problems in Shoulder Surgery. Philadelphia, PA: Lippincott-Raven; 1996:47-63.

Sanchez-Sotelo J, Sperling JW, Rowland CM, Cofield RH. Instability after shoulder arthroplasty: results
of surgical treatment. J Bone Joint Surg Am. 2003 Apr;85-A(4):622-31. PubMed PMID: 12672836.

Moeckel BH, Altchek DW, Warren RF, Wickiewicz TL, Dines DM. Instability of the shoulder after
arthroplasty. J Bone Joint SurgAm. 1993 Apr;75(4):492-7. PubMed PMID: 8478377.

Miller BS, Joseph TA, Noonan TJ, Horan MP, Hawkins RJ. Rupture of the subscapularis tendon
after shoulder arthroplasty: diagnosis, treatment, and outcome. J Shoulder Elbow Surg. 2005 Sep-
Oct;14(5):492-6. PubMed PMID: 16194740.

(;I 2014 American Academy of Orthopaedic Surgeons 2014 Shoulder and Elbow Self-Assessment Examination
2014 Shoulder and Elbow Self-Assessment Examination Answer Book • 93

Figure 93

Question 93
Figure 93 is the radiograph of a 72-year-old woman. Treatment includes fixation of the ulna. What options
are recommended for the radius?

1. Radial head replacement to restore radiocapitellar contact


2. Radial head excision because there is no risk for posterolateral instability
3. Percutaneous fixation to avoid the risk for stiffness after surgery
4. Allograft reconstruction to prevent capitellar erosion

PREFERRED RESPONSE: 1

Prosthetic replacement is an appropriate option in cases of a nonreconstructable fracture to restore the


radiocapitellar contact. Most complex fractures are associated with instability; therefore, it is advisable
to consider open reduction and internal fixation or radial head replacement when the injury involves a
dislocation or fracture of the ulna. Simple radial head excision may be a viable option for a com.minuted
fracture without instability or associated ulnar fracture. When the radial head is replaced, caution must be
exercised to avoid overstuffing the joint because this can lead to stiffness from impingement, capitellar
erosion, loss of flexion, or synovitis.

IO 2014 American Academy of Orthopaedic Surgeons 2014 Shoulder and Elbow Self-Assessment Examination
94 • American Academy of Orthopaedic Surgeons

RECOMMENDED READINGS
Ring D. Radial head fractures. In: Galatz L, ed. Orthopaedic Knowledge Update Shoulder and Elbow 3.
Rosemont, IL: American Academy of Orthopaedic Surgeons; 2008:497-506.

Rosenblatt Y, Athwal GS, Faber KJ. Current recommendations for the treatment ofradial head fractures.
Orthop Clin North Am. 2008 Apr;39(2):173-85, vi. doi: 10.1016/j.ocl.2007.12.008. Review. PubMed
PMID: 18374808.

Duckworth AD, Watson BS, Will EM, Petrisor BA, Walmsley PJ, Court-Brown CM, McQueen
MM. Radial head and neck fractures: functional results and predictors of outcome. J Trauma. 2011
Sep;71(3):643-8. doi: 10.1097/fA.0b013e3181f8fa5f. PubMed PMID: 21248649.

Herbertsson P, Josefsson PO, Hasserius R, Karlsson C, Besjakov J, Karlsson M; Long-Term Follow-Up


Study. Uncomplicated Mason type-II and III fractures of the radial head and neck in adults. A long-term
follow-up study. J Bone Joint SurgAm. 2004 Mar;86-A(3):569-74. PubMed PMID: 14996884.

Figure 94

CLINICAL SITUATION FOR QUESTIONS 94 THROUGH 96


Figure 94 is the anteroposterior radiograph of a 75-year-old woman who has a 5-year history of
progressive pain, crepitus, and loss of motion in her shoulder. She had a rotator cuff repair 10 years ago.
Examination reveals 60 degrees of active forward elevation and 20 degrees of external rotation with
her arm at her side. Passively she can be brought to 160 degrees of forward elevation and 90 degrees of
external rotation with her arm at her side. A glenohumeral joint injection with local anesthetic eliminated
pain, but there is no observed change in active motion.

IO 2014 American Academy of Orthopaedic Surgeons 2014 Shoulder and Elbow Self-Assessment Examination
2014 Shoulder and Elbow Self-Assessment Examination Answer Book• 95

Question 94
Based upon the information provided, you should recommend

1. total shoulder arthroplasty (TSA).


2. arthroscopic rotator cuff repair.
3. arthroscopic debridement.
4. reverse total shoulder arthroplasty (rTSA).

PREFERRED RESPONSE: 4

Question 95
Your treatment decision is the best option because the

1. prosthesis is designed to convert the translational force of the deltoid to rotational motion.
2. use of an all-polyethylene glenoid component will reduce risk for developing glenoid pain
after humeral head arthroplasty.
3. poor motion is a function of synovitis.
4. weakness is generated from pain.

PREFERRED RESPONSE: 1

Question 96
A common postoperative radiographic observation associated with your surgery in an asymptomatic
patient is

1. implant fracture.
2. suture anchor dislodgement.
3. scapular notching.
4. acromial fracture.

PREFERRED RESPONSE: 3

© 2014 American Academy of Orthopaedic Surgeons 2014 Shoulder and Elbow Self-Assessment Examination
96 • American Academy of Orthopaedic Surgeons

DISCUSSION FOR QUESTIONS 94 THROUGH 96


The patient has anterior superior instability. This condition is caused by a combination of rotator cuff
insufficiency and loss of coracoacromial arch integrity. The only known way to restore shoulder function
in this scenario is to implant a rTSA. The device works by converting the translational force of the deltoid
into a rotational force, resulting in restoration of forward elevation of the shoulder. Performing a rotator
cuff repair or arthroscopic debridement will not address this biomechanical problem. TSA will also not
change this biomechanical problem. The poor motion and function are not a result of synovitis or pain
because an injection with local anesthetic has eliminated the pain and serves as a useful test to determine
ifrTSA is the only viable solution. If the patient can achieve near-normal function with a local anesthetic
challenge, rTSA is overtreatment. Scapular notching is a long-term concern for implant longevity because
it represents bone loss under the baseplate of the glenoid component. This loss of support can lead to
catastrophic failure of the device. Implant fracture, acromial fracture, and dislodgement of suture anchors
are not likely to be asymptomatic in a non-Charcot joint.

RECOMMENDED READINGS FOR QUESTIONS 94 THROUGH 96


Melis B, Defranco M, Ladermann A, Mole D, Favard L, Nerot C, Maynou C, Walch G. An evaluation of
the radiological changes around the Grammont reverse geometry shoulder arthroplasty after eight to 12
years. J Bone Joint Surg Br. 2011 Sep;93(9):1240-6. PubMed PMID: 21911536.

Leung B, Horodyski M, Struk AM, Wright TW. Functional outcome of hemiarthroplasty compared with
reverse total shoulder arthroplasty in the treatment of rotator cuff tear arthropathy. J Shoulder Elbow Surg.
2012 Mar;21(3):319-23. Epub 2011 Aug 26. PubMed PMID: 21872496.

Cheung E, Willis M, Walker M, Clark R, Frankie MA. Complications in reverse total shoulder
arthroplasty. J AmAcad Orthop Surg. 2011 Jul;19(7):439-49. Review. PubMedPMID: 21724923.

Walker M, Brooks J, Willis M, Frankie M. How reverse shoulder arthroplasty works. Clin Orthop Relat
Res. 2011 Sep;469(9):2440-51.Review. PubMed PMID: 21484471.

Sadoghi P, Vavken P, Leithner A, Hochreiter J, Weber G, Pietschmann MF, Muller PE. Impact of
previous rotator cuff repair on the outcome of reverse shoulder arthroplasty. J Shoulder Elbow Surg. 2011
Oct;20(7):1138-46. Epub 2011 Mar 30. PubMed PMID: 21454102.

Nam D, Kepler CK, Neviaser AS, Jones KJ, Wright TM, Craig EV, Warren RF. Reverse total shoulder
arthroplasty: current concepts, results, and component wear analysis. J Bone Joint SurgAm. 2010 Dec;92
Suppl 2:23-35. PubMed PMID: 21189245.

END OF SERIES

© 2014 American Academy of Orthopaedic Surgeons 2014 Shoulder and Elbow Self-Assessment Examination
2014 Shoulder and Elbow Self-Assessment Examination Answer Book • 97

Question 97
A 36-year-old woman dislocated her elbow 6 months ago. The elbow was congruently reduced and
rehabilitated. She continues to have a sense of painful clunking in her elbow when she pushes up from a
chair with forearm supination, but not pronation. What structure did not heal properly?

1. Posterior band of the medial collateral ligament


2. Anterior band of the medial collateral ligament
3. Radial collateral ligament
4. Lateral ulnar collateral ligament

PREFERRED RESPONSE: 4

DISCUSSION
The patient is showing signs of posterolateral rotatory instability after elbow dislocation. The lateral ulnar
collateral ligament is responsible for stabilizing the elbow against this type of instability. The posterior and
anterior bands of the medial collateral ligament are primarily resistors ofvalgus load in elbow extension
and flexion, respectively. The radial collateral ligament does not control the posterolateral rotatory
instability described.

RECOMMENDED READINGS
O'Driscoll SW, Morrey BF, Korinek S, An KN. Elbow subluxation and dislocation. A spectrum of
instability. Clin Orthop Relat Res. 1992 Jul;(280):186-97. PubMed PMID: 1611741.

O'Driscoll SW, Bell DF, Morrey BF. Posterolateral rotatory instability of the elbow. J Bone Joint SurgAm.
1991 Mar;73(3):440-6. PubMed PMID: 2002081.

Question 98
What complication following total elbow arthroplasty poses more risk for a 60-year-old man with
osteoarthritis than for a man of the same age with rheumatoid arthritis?

1. Aseptic loosening of a linked implant


2. Instability of an unlinked implant
3. Triceps rupture
4. Wound dehiscence

PREFERRED RESPONSE: 1

0 2014 American Academy of Orthopaedic Surgeons 2014 Shoulder and Elbow Self-Assessment Examination
98 • American Academy of Orthopaedic Surgeons

DISCUSSION
Patients with elbow osteoarthritis tend to be active and are often involved in manual occupations that
place greater demands on a total elbow implant. Such patients are most often treated with nonprosthetic
options because of concerns about prosthetic longevity. As a result, few cases of primary osteoarthritis are
included in published studies. However, complications such as stem fracture and aseptic loosening appear
to be more common in this population than in any other subgroup, including revision patients. The poor
soft-tissue quality associated with rheumatoid arthritis leads to a high-risk ligamentous attenuation and is
a general contraindication to use of an unlinked implant. The same poor soft tissue leads to a higher rate of
triceps insufficiency and wound dehiscence.

RECOMMENDED READINGS
Kozak TKW, Adams RA, Morrey BF. Total elbow arthroplasty for primary osteoarthritis. In: Morrey BF,
Sanchez-Sotelo J, eds. The Elbow and Its Disorders. Philadelphia, PA: WB Saunders; 2009:843-848.

Gill DRJ, Morrey BF, Adams RA. Linked total elbow arthroplasty in patients with rheumatoid arthritis.
Total elbow arthroplasty for primary osteoarthritis. In: Morrey BF, Sanchez-Sotelo J, eds. The Elbow and
Its Disorders. Philadelphia, PA: WB Saunders; 2009:782-791.

Gramstad GD, Galatz LM. Management of elbow osteoarthritis. J Bone Joint Surg Am. 2006
Feb;88(2):421-30. Review. PubMed PMID: 16452758.

Gill DR, Morrey BF. The Coonrad-Morrey total elbow arthroplasty in patients who have rheumatoid
arthritis. A ten to fifteen-year follow-up study. J Bone Joint Surg Am. 1998 Sep;80(9):1327-35. PubMed
PMID: 9759818.

© 2014 American Academy of Orthopaedic Surgeons 2014 Shoulder and Elbow Self-Assessment Examination
2014 Shoulder and Elbow Self-Assessment Examination Answer Book • 99

Figure 99a Figure 99b

Question 99
Figure 99a is the radiograph of a 48-year-old woman 8 months after initial treatment of an injury. She
initially was placed in a sling and progressive rehabilitation followed. She now has refractory pain but
normal range of movement and strength. The current radiograph is shown in Figure 99b. The most
appropriate next treatment step is

1. Resumption of sling immobilization


2. Open reduction and internal fixation
3. Application of a bone stimulator and rest from exacerbating activities
4. Application of a figure-of-8 brace

PREFERRED RESPONSE: 2

DISCUSSION
The radiographs reveal an atrophic nonunion of the midshaft of the clavicle. The treatment of acute,
displaced midshaft clavicle fractures in adults continues to evolve, with several reports advocating early
surgical intervention. Although many fractures heal, symptomatic delayed unions or nonunions occur and
may eventually require treatment. In this case, further sling immobilization or use of a figure-of-8 brace
is unlikely to lead to fracture consolidation at 8 months after the injury. Although use of an electrical
bone stimulator may be attractive, there is no conclusive data suggesting its efficacy in promoting healing
of a displaced clavicular nonunion. Most authors advocate treatment with open reduction internal plate
fixation. Controversy exists as to the need for allograft or autograft bone augmentation.

RECO:MMENDED READINGS
van der Meijden OA, Gaskill TR, Millett PJ. Treatment of clavicle fractures: current concepts review. J
Shoulder Elbow Surg. 2012 Mar;21(3):423-9. doi: 10.1016/j.jse.2011.08.053. Epub 2011 Nov 6. Review.
PubMed PMID: 22063756.

Khan LA, Bradnock TJ, Scott C, Robinson CM. Fractures of the clavicle. J Bone Joint Surg Am. 2009
Feb;91(2):447-60. doi: 10.2106/JBJS.H.00034. Review. PubMed P:MID: 19181992.

Endrizzi DP, White RR, Babikian GM, Old AB. Nonunion of the clavicle treated with plate fixation:
a review of forty-seven consecutive cases. J Shoulder Elbow Surg. 2008 Nov-Dec;l7(6):951-3. doi:
10.1016/j.jse.2008.05.046. Epub 2008 Sep 20. PubMed PMID: 18805024.

Cl 2014 American Academy of Orthopaedic Surgeons 2014 Shoulder and Elbow Self-Assessment Examination
100 • American Academy of Orthopaedic Surgeons

Question 100
A 75-year-old woman sustained a 4-part fracture dislocation of the proximal humerus with a comminuted
humeral head. You decide to perform a reverse total shoulder replacement because of her age and activity
level. This will be your first reverse total shoulder replacement. It is common practice in your hospital for
an industry representative to be present when new implants are brought into the operating room. What
information are you required to disclose?

1. This is an experimental procedure.


2. You have no :financial relationship with the implant company.
3. There will be an implant company representative in the room.
4. The hospital will lose money because of the cost of the implant.

PREFERRED RESPONSE: 3

DISCUSSION
Current recommendations from the American Orthopaedic Association Orthopaedic Institute of Medicine
are that the patient be notified if an industry representative is going to be present in the operating room.
This surgery is not experimental for this indication, and Medicare currently covers the surgery for patients
with appropriate indications. Court cases have demonstrated that surgeon-related factors can be litigated
(such as surgeon experience), but there are no current requirements to disclose this. Surgeons are not
required to disclose cost and compensation information to their patients.

RECOMMENDED READINGS
Report from the task force on surgeon-industry relationships in the discipline of orthopaedic surgery.
American Orthopaedic Association Orthopaedic Institute of Medicine Task Force on Surgeon-Industry
Relationships in the Discipline of Orthopaedic Surgery. J Bone Joint SurgAm. 2012 Jun 20;94(12):e89.
No abstract available. PMID: 22717841.

Bal BS, Choma TJ. Clin Orthop Relat Res. What to disclose? Revisiting informed consent. 2012
May;470(5):1346-56. PMID: 22215479.

Cl 2014 American Academy of Orthopaedic Surgeons 2014 Shoulder and Elbow Self-Assessment Examination
2014 Shoulder and Elbow Self-Assessment Examination Answer Book • 10I

RESPONSES FOR QUESTIONS 101 THROUGH 104


1. Arthroscopic or open debridement and capsular release
2. Interposition arthroplasty
3. Ulnohumeral arthrodesis
4. Linked total elbow arthroplasty (TEA)
5. Unlinked TEA

What surgical procedure listed above is most associated with the conditions defined below?

Question 101
This procedure has implant or graft loosening as the main postsurgical concern.

PREFERRED RESPONSE: 4

Question 102
This procedure is associated with progressive resorption of distal humeral condyles after surgery.

PREFERRED RESPONSE: 2

Question 103
A 50-year-old woman with poorly controlled rheumatoid arthritis has advanced destruction of the right-
dominant elbow.

PREFERRED RESPONSE: 4

Question 104
A 41-year-old man who is a carpenter has moderate posttraumatic arthritis and pain in terminal extension
affecting function.

PREFERRED RESPONSE: 1

tt:l 2014 American Academy of Orthopaedic Surgeons 2014 Shoulder and Elbow Self-Assessment Examination
102 • American Academy of Orthopaedic Surgeons

DISCUSSION FOR QUESTIONS 101 THROUGH 104


Linked prosthetic TEA has been a common choice for surgical treatment of elbow arthritis, and reports
document good results in many patients. However, load on the bearing surfaces and on the implant/
cement/ bone interface are sources of failure, particularly in younger and higher-demand patients. The
use of nonprosthetic options such as arthroscopic debridement or interposition arthroplasty is advocated
in this population. Unlinked arthroplasties have been developed in an effort to reduce stem loosening by
decreasing the constraint of the articulation.

In contrast, patients with rheumatoid arthritis often have attenuated ligamentous support and may develop
instability with the same unlinked implants. Patients with rheumatoid arthritis also typically have lower
demands because of polyarticular disease. Linked TEA has been shown to have similar survival as total
hip replacement in this population.

Bone resorption is an occasional complication of interposition arthroplasty. This can lead to subluxation
and poor outcome, particularly if it occurs more on one side than the other.

Unlinked TEA would be considered in severe, concentric arthritis with intact ligamentous support in
young, active patients. Ulnohumeral arthrodesis has poor functional outcome and rarely is considered a
salvage procedure.

RECOM11ENDED READINGS FOR QUESTIONS 101 THROUGH 104


Kozak TKW, Adams RA, Morrey BF. Total elbow arthroplasty for primary osteoarthritis. In: Morrey BF,
Sanchez-Sotelo J, eds. The Elbow and Its Disorders. Philadelphia, PA: WB Saunders; 2009:843-849.

Gill DRJ, Morrey BF, Adams RA: Linked total elbow arthroplasty in patients with rheumatoid arthritis.
In: Morrey BF, Sanchez-Sotelo J, eds. The Elbow and Its Disorders. Philadelphia, PA: WB Saunders;
2009:782-791.

Gramstad GD, Galatz LM. Management of elbow osteoarthritis. J Bone Joint Surg Am. 2006
Feb;88(2):421-30. Review. PubMed PMID: 16452758.

Grishin JG, Goncharenko Iv, Kozhin NP, SarkisyanAG, Gobulev AG, DevisAE. Restoration of the
function of the cubitaljoint in extensive defects of bones and soft tissues using endoprosthesis and free
skin grafts. Acta Chir Plast. 1989;31(3):143-7. PubMed PMID: 2481919.

Gill DR, Morrey BF. The Coonrad-Morrey total elbow arthroplasty in patients who have rheumatoid
arthritis. A ten to fifteen-year follow-up study. J Bone Joint SurgA.m. 1998 Sep;80(9):1327-35. PubMed
PMID: 9759818.

END OF SERIES

Cl 2014 American Academy of Orthopaedic Surgeons 2014 Shoulder and Elbow Self-Assessment Examination
2014 Shoulder and Elbow Self-Assessment Examination Answer Book• 103

Figure 105a Figure 105b

Question 105
Figures 105a and 105b are the radiograph and MRI scan of a 45-year-old woman with :fibromyalgia that
causes chronic neck and scapula pain. She has had new-onset lateral shoulder pain for 1 year. She has
tenderness throughout her shoulder, back, and neck; a positive Hawkins impingement sign; and pain
with resisted elevation. She tried physical therapy for 12 weeks and the pain is worse. What is the next
appropriate treatment step for her shoulder pain?

1. Subacromial injection
2. Referral to pain management
3. Arthroscopic removal of a calcium deposit
4. Arthroscopic subacromial decompression

PREFERRED RESPONSE: 1

DISCUSSION
The radiographs show a calcium deposit in the supraspinatus tendon. The MRI scan shows a small black
signal in the supraspinatus tendon without a tear of the tendon. These findings are consistent with calcific
tendonitis. A subacromial injection will help distinguish between the pain from the calcific tendonitis and
the chronic fibromyalgia. Pain management is an option after an injection is done for diagnostic purposes.
Surgery should not be considered until the diagnosis is confirmed because it will not be successful if the
shoulder pain is attributable to the patient's fibromyalgia.

RECOMMENDED READINGS
Hurt G, Baker CL Jr. Calcific tendinitis of the shoulder. Orthop Clin North Am. 2003 Oct;34(4):567-75.
Review. PubMed PMID: 14984196.

Porcellini G, Paladini P, Campi F, Paganelli M. Arthroscopic treatment of calcifying tendinitis of the


shoulder: clinical and ultrasonographic follow-up :findings at two to five years. J Shoulder Elbow Surg.
2004 Sep-Oct;13(5):503-8. PubMed PMID: 15383805.

IC 2014 American Academy of Orthopaedic Surgeons 2014 Shoulder and Elbow Self-Assessment Examination

You might also like